Breast/Endocrine System Flashcards

1
Q

Three months ago you started Mary, aged 73, on residronate 35mg weekly, after she was diagnosed with osteoporosis confirmed with bone mineral density (BMD). Her presenting symptom at that time was back pain. Examination revealed height decrease and mild kyphosis, as well as tenderness over thoracic vertebrae. Despite being on residronate, her symptoms persisted and she also developed pain over new areas of her thoracic vertebrae. A new X-ray established new osteoporotic fractures. Which one of the following would be the most appropriate management option for her?

A. Continue residronate at the same dose.
B. Switch to alendronate.
C. Increase the dose of residronate.
D. Switch to zoledronic acid.
E. Switch to teriparatide.

A

E. Switch to teriparatide.

Bisphosphonates are currently the most commonly anti-resorptive agents prescribed as first-line treatment for most of osteoporotic patients. Of this drug family, alendronate (10mg/day or 70mg/week, orally), residronate (5mg/day or 35mg/week, orally) and zoledronic acid are available in Australia.

Bisphosphonates should be used for at least 12 months before their efficacy on treatment of osteoporosis is assessed. However, in cases where the patient suffers two or more minimal trauma fractures despite being on sufficient doses of an anti-resorptive drug, e.g. bisphosphonates, commencement of teriparatide is justified as the most appropriate option .

Teriparatide is the synthetic parathyroid hormone that predominantly acts by increasing the osteoblasts (bone-forming cells) and by inducing new osteoblasts formation. This drug is costly and at least 18 months of continuous use is needed effectiveness. Based on these, this medication is only reimbursed by the PBS for patients with severe osteoporosis and very high risk of fractures who have:
* A BMD T-score of =<-3
OR
* had two or more minimal trauma fractures
OR
* experienced at least one symptomatic new fracture after at least 12-months of continuous therapy with an anti-resorptive agent at adequate doses. [? Mary’s case]

Option A: While Mary has developed new osteoporotic fractures due to severe osteoporosis, continuation of the same agent is not a wise management.

Option B and D: Switching to other bisphosphonates such as alendronate or zoledronic acid does not add any benefit, as all members of this drug family have almost the same effectiveness.

Option C: Mary is already on the recommended weekly dose for residronate and increasing the dose of residronate does not seem to benefit her.

How well did you know this?
1
Not at all
2
3
4
5
Perfectly
2
Q

Jane, a 65-year-old patient of yours, is being assessed for osteoporosis. A Dual Energy X-ray Absorptiometry has revealed T-scores of -2.5 and -2.7 for the femoral neck and the vertebral column, respectively. She was diagnosed with the cancer of her right breast 6 years ago for which she underwent right mastectomy, chemotherapy and radiation therapy. In addition to advice regarding calcium and vitamin D, which one of the following medications would be the best option for treatment of her osteoporosis?

A. Raloxifene.
B. Alendronate.
C. Teriparatide.
D. Strontium ranelate.
E. Hormone replacement therapy (HRT).

A

B. Alendronate.

According to WHO criteria and based on Jane’s T-scores at vertebra and femoral neck, she is suffering from osteoporosis. Currently, bisphosphonates are the first-line treatment option for primary and secondary treatment of vertebral and non-vertebral fractures due to osteoporosis. Alendronate, residronate and zoledronic acid are the available bisphosphonates in Australia .

Option A: Raloxifene is a selective estrogen receptor modulator (SERM) reimbursed by the Pharmaceutical Benefit Scheme (PBS) for treatment of postmenopausal osteoporosis. SERMs also have a preventive effect on breast cancer; therefore, an appropriate option for post- menopausal women with family or personal history of breast cancer in whom vertebral fracture due osteoporosis is a concern. Although there is excellent evidence (Grade A) that raloxifene reduces the risk of vertebral fractures, limited evidence support its effect on non-vertebral fractures. Jane is osteoporotic in femur as well; hence raloxifene would not an appropriate first-line option for her.

Option C: Teriparatide is the synthetic parathyroid hormone that predominantly acts by increasing the osteoblasts (bone-forming cells) and by inducing new osteoblasts formation. This drug is costly and at least 18 months of continuous use is needed effectiveness. Based on these, this medication is only reimbursed by the PBS for patients with severe osteoporosis and very high risk of fractures who have:
* A BMD T-score of =<-3
OR
* had two or more minimal trauma fractures
OR
* experienced at least one symptomatic new fracture after at least 12-months of continuous therapy with an anti-resorptive agent at adequate doses.

Jane does not any of the above criteria to meet the need for teriparatide.

NOTE - BMD starts declining approximately 12 months after cessation of therapy with teriparatide; hence, continuation of therapy with an anti-resorptive, generally a bisphosphonates.

Option D: Strontium ranelate is an effective second-line option for reducing the risk of further osteoporotic fractures in postmenopausal women with prevalent fractures. This agent should not be used in patients with previous or clinically active cardiovascular disease or uncontrolled hypertension and should only be used when other drug classes for treatment of osteoporosis are not appropriate. Strontium ranelate is prescribed at a dose of 2 grams per day.

Option E: Breast cancer is estrogen dependent; therefore, prescribing estrogen for patients with a personal or strong family history of breast cancer is not an appropriate option. Estrogen is an option (available on PBS) for the prevention and treatment of osteoporosis in women who are near or at menopause, in particular for those patients suffering from menopausal vasomotor symptoms, e.g. hot flushes. When considering this therapy, potential adverse effects i.e. increased risk of thromboembolism, cardiovascular diseases and breast cancer should be weighed and discussed with the patient.

How well did you know this?
1
Not at all
2
3
4
5
Perfectly
3
Q

A 54-year-old woman presents to your clinic for advice regarding osteoporosis. You order a DEXA scan showing a T-score of -1. Which one of the following is the most appropriate management for this patient?

A. High-calcium diet.
B. Bisphosphonate.
C. Calcium and vitamin D supplement.
D. Adequate exercise.
E. Exposure to sunlight.

A

A. High-calcium diet.

A T-score of -2.5 of less indicates osteoporosis.
T-scores between -1 to -2.5 suggests osteopenia.

A T score of -1 or above is considered normal. So this woman is neither osteopenic nor osteoporotic. For postmenopausal woman without evident bone loss, maintaining a high-calcium diet is the most appropriate advice for prevention of osteoporosis (grade A recommendation). Diet is the preferred source of calcium, and patients should aim for a total daily calcium intake of 1300 mg. Dairy products are rich in calcium and one of the best sources.

Option B: Bisphosphonates are used for treatment of established osteoporosis or osteopenia in ceratin patient groups . This woman does not have osteoporosis.

Option C: Calcium supplementation (with or without vitamin D, depending on the patient’s vitamin D status) is used when adequate calcium intake cannot be ensured solely through diet.

Option D: Exercise may have additional benefits including weight control, reduction of blood pressure, pain relief and improvement in quality of life. Exercise can also be beneficial for improving balance as a part of a ‘falls reduction program’. Particularly in patients diagnosed with osteoporosis, supervision by a physiotherapist, exercise physiologist or other appropriately trained and qualified health professional is recommended. Overall, moderate exercise against gravity, such as brisk walking for 30 minutes 4 times a week, jogging or tennis may make a small contribution to retarding bone loss. Non-weight bearing exercises such as swimming, cycling, etc are not effective in bone loss prevention.

Option E: Exposure to sunlight is necessary for vitamin D production. Vitamin D is necessary for calcium absorption through the gut.

How well did you know this?
1
Not at all
2
3
4
5
Perfectly
4
Q

Which one of the following breast diseases can be caused by vitamin A deficiency?

A. Fibroadenoma.
B. Intraductal papilloma.
C. Periductal mastitis.
D. Ductal carcinoma in situ.
E. Paget disease.

A

C. Periductal mastitis.

Of the given options, periductal mastitis is found to be caused by vitamin A deficiency.

Periductal mastitis, also known as subareolar abscess, squamous metaplasia of lactiferous ducts, or Zuska disease is an inflammatory condition of the subareolar ducts. Periductal mastitis primarily affects young women, but can occur in men as well. Periductal mastitis is a separate entity from duct ectasia, which usually affects older women and is characterized by dilated ducts and sticky toothpaste-like dischagre.

The cause of periductal mastitis is unknown. However, vitamin A deficiency and smoking have been implicated as potential causes.

The majority of patients who get periductal mastitis are smokers. It is postulated that smoking leads to or is associated with damage
to subareolar ducts with subsequent tissue necrosis and later infection. The toxic substances in cigarette smoke may damage the ducts directly, or there may be a localized hypoxic effect.

There is increasing evidence that vitamin A (or retinoids) have a significant effect on mammary duct epithelial cell proliferation and differentiation. Vitamin A deficiency impairs blood clearance of bacteria and results in decreased phagocytic activity in vitro.

How well did you know this?
1
Not at all
2
3
4
5
Perfectly
5
Q

A 12-year-old boy is brought to the emergency department with loss of consciousness. His blood glucose on arrival is 2.5 mmol/L. His past medical history is significant for type I diabetes mellitus, for which he is on insulin. Which one of the following is the most appropriate management?

A. Dextrose 5% intravenously.
B. Dextrose 10% intravenously.
C. Dextrose 50% intravenously.
D. Normal saline.
E. Oral sweet juice.

A

B. Dextrose 10% intravenously.

In children with type I diabetes mellitus, hypoglycemia is the most common encountered complication. Hypoglycemia may lead to seizures and loss of consciousness.

In childhood (other than neonatal period), hypoglycaemia is defined as a blood glucose level of < 2.6 mmol/L. The most appropriate treatment of severe hypoglycemia when the patient is unconscious and unable to take oral glucose is with a bolus of intravenous dextrose 10%, 2.5 to 5 mL/Kg followed by 0.03 to 0.05 mL/Kg/minute until the patient is stable.

In adults with hypoglycemia, 50% glucose solution is used for treatment of severe hypoglycemia. This solution is not recommended for children because it can result in serum hyperosmolarity and death.

In a conscious and cooperative child, oral route is preferred.

How well did you know this?
1
Not at all
2
3
4
5
Perfectly
6
Q

A 38-year old lady presents to your pratice with fatigue, lethargy and weight gain of 5 kg over the past few months. She also has irregular heavy periods. She feels flushed and tremulous. She is hypertensive, and her blood sugar is 10 mmol/L. Which one of the following is the most appropriate choice of investigation?

A. Thyroid function tests.
B. 24-hour urine cortisol level.
C. Serum FSH and LH.
D. Serum FSH and estradiol.
E. Serum prolactin level.

A

B. 24-hour urine cortisol level.

This patient has clinical features suggestive of Cushing’s syndrome.

Cushing’s syndrome in women may present with:
* Anxiety
* Tremulousness
* Weight gain
* Severe fatigue
* Menstrual irregularities
* Hypertension
* Hyperglycemia
* Thin easily-bruising skin
* Purplish pink striae may be present on buttocks, thighs, abdomen and breast
* Proximal muscle weakness

When Cushing syndrome is suspected, overnight dexamethasone (1 mg) challenge test is performed as screening. Patients in whom cortisol level fails to become suppressed should undergo high-dose dexamethasone suppression test. If high-dose dexamethasone fails to suppress serum cortisol levels, of 24-hour urine cortisol level should be measured as the most definitive diagnosis.

How well did you know this?
1
Not at all
2
3
4
5
Perfectly
7
Q

A 57-year-old woman presents to your clinic complaining of low back pain. She has a history of breast cancer, successfully treated 4 years
ago. She does not go out much because she is afraid of developing melanoma. A Dexa scan shows a T score of -2.8 g/cm2. An X-ray reveals 2 vertebral fractures. Which one of the following would be the most appropriate next step in management?

A. Alendronate.
B. Raloxifene.
C. Vitamin D.
D. Calcium.
E. Strontium ranelate.

A

C. Vitamin D.

With two fractured vertebrae and a T-score of -2.8, this patient is suffering from osteoporosis. Medical treatment is indicated in the following patient groups:
* Those with a minimal trauma fracture
* Those aged 70 years or over
* Those with a T-score of –2.5 or lower
* Those currently on prolonged (at least 3 months) high dose corticosteroid treatment (at least 7.5 mg/day prednisolone or equivalent) and with a T-score of –1.5 or lower.

There are different options for medical prevention and treatment of osteoporosis such as bisphosphonates (e.g. alendronate, risedronate), estrogen receptor modulators (e.g. raloxifene), strontium ralenate, and teriparatide.

Supplementation of calcium and vitamin D should be considered for all patients, as studies suggest that the optimal efficacy of medical treatment, to a considerable extend, depends on the availability of these two.

Bisphosphonates are always first-line medications in postmenopausal women (even those with personal history of breast cancer).

Whenever bisphosphonates are considered for treatment, close attention should be paid to vitamin D status and its deficiency be corrected prior to starting treatment. Achieving a 25-hydroxy vitamin D plasma level of 50nmol/L is the target before the patient is started on bisphosphonates.

Vitamin D supplementation is indicated for the following patient groups:
* Those with proven vitamin D deficiency
* Institutionalized or housebound people
* Women who are shrouded for cultural reasons

Since this woman does not expose herself to sunlight due to fear of developing melanoma, she should be categorized as housebound and vitamin D deficient because exposure to sunlight is the most significant source of vitamin D production in the body. She definitely needs alendronate as first-line medical treatment for her osteoporosis, but not before her vitamin D deficiency is corrected to a level of 50nmol/L or above.

How well did you know this?
1
Not at all
2
3
4
5
Perfectly
8
Q

Jennifer is a 67-year-old female patient of yours in the GP clinic you work. She has had type 2 diabetes mellitus for the past 10 years. Despite advice and treatment, she has poor glycemic control with the last HbA1C ordered 2 months ago being 10.7%. She also has hypercholesterolemia and hypertension for which she is on treatment. Which one of the following would be the most appropriate screening advice for her regarding retinopathy?

A. Review by an ophthalmologist every 2 years.
B. Yearly review by an optometrist.
C. Review by ophthalmologist every 5 years.
D. Tight glycemic control followed by retinal photography.
E. Review by an optometrist every 2 years.

A

B. Yearly review by an optometrist.

Type 2 diabetes mellitus is associated with several complications including, cardiovascular disease, peripheral neuropathy, diabetic nephropathy, diabetic foot, and diabetic retinopathy. There are recommendations for appropriate screening of patients with type 2 diabetes for early detection and treatment of such complications.

Current guidelines by the National Health and Medical Research Council (NHMRC) recommends the following:
* Diabetic retinopathy
All patients with type 2 diabetes should have dilated fundus examination and visual acuity assessment at the diagnosis of diabetes and at least every two years. Higher risk patients (e.g., longer duration of diabetes, or poor glycemic control, blood pressure or blood lipid control) who do not have diabetic retinopathy at least every year.

  • Diabetic peripheral neuropathy
    All patients should be screened for distal symmetric polyneuropathy starting at diagnosis of type 2 diabetes and at least annually thereafter, using simple clinical tests.
  • Diabetic nephropathy
    Kidney status in people with type 2 diabetes should be assessed by yearly screening for albuminuria (note that dipstick urine test is not adequate to identify albuminuria) and estimated glomerular filtration rate (eGFR)
  • Glycemic control:
    Apart from routine screening of blood sugar by the patient, HbA1C should be tested every 3-6 months
  • Lipids:
    If the patient has low CVD risk, these tests can be performed every three years. More frequent testing can be justified if the clinical situation varies or if therapeutic changes have been made e.g., a change in type or dose of the lipid lowering agent. Some guidelines suggest yearly testing of lipids when the patient is deemed to be at clinically high risk.
  • Other tests:
    Based upon a clinical risk assessment, individually assess the need for further investigations such as liver enzyme abnormalities for hepatic steatosis.

Jennifer has long-standing diabetes with poor glycemic control. Besides, she has hypertension and hyperlipidemia as well. These, collectively, put her at higher risks of diabetic retinopathy. For that, she needs to be screened yearly by an optometrist, ophthalmologist or another health professional with enough expertise in the field. The assessment can be done by pupillary dilation and ophthalmoscopy or retinal photography.

Options suggesting 2-yearly screening (options A and E) could be the correct answers for people with diabetes an no increased risk of retinopathy. Any option suggesting longer than 2 years, e.g., 5-yearly (option C) is wrong in all circumstances and for all patients with type 2 diabetes.

Option D: Although tight glycemic control should be advised and emphasized on, screening is not dependent on it and screening should be advised and carried out regardless of the glycemic control.

How well did you know this?
1
Not at all
2
3
4
5
Perfectly
9
Q

Which one of the following is least likely to be found on clinical examination of patient with thyrotoxicosis?

A. Insomnia.
B. Fine tremors.
C. Painful hip.
D. Atrophy of interosseous muscles.
E. Wide pulse pressure.

A

C. Painful hip.

Thyrotoxicosis can be caused by the folloiwng four conditions, of which the first two are the most common causes in Australia (percentages for Australian population):
1. Graves’ Disease (70%)
2. Toxic multinodular goitre (15%)
3. Toxic adenoma (5%)
4. Thyroiditis (5%)

Symptoms of thyrotoxicosis depend on the severity and duration of the disease, the specific cause of the thyrotoxicosis, and the age of the patient. Excessive thyroid hormone affects virtually all body systems through the stimulation of metabolic processes and activation of the sympathetic nervous system.

In general, the clinical features include:
* Weight loss (weight gain in 10% due to increased appetite)
* Heat intolerance
* Palpitations
* Breathlessness
* Heart failure in the elderly
* Irritability and insomnia (option A)
* Nervousness, anxiety
* Tiredness and lethargy
* Diarrhea
* A fine tremor (option B), with proximal muscle weakness (unable to climb stairs or rise from a sitting position)
* Sweating
* Tachycardia
* Goitre
* Vitiligo and alopecia
* Pretibial myxedema: skin thickening like orange peel, common over the shins
* Wide pulse pressure (option E)
* Eye changes - related to sympathetic stimulation and (in Graves) to antibody reactions. Thyroid eye disease of Graves’ disease may present with proptosis, eye muscle involvement, corneal involvement, and optic nerve compression. lid retraction and lid lag are common to all causes of thyrotoxicosis.

The muscular symptoms of thyrotoxicosis vary from mild myasthenia to profound muscular weakness and atrophy, especially of proximal muscle groups. This weakness forms the basis of a useful clinical test. If a thyrotoxic patient seated in a chair is asked to hold one leg out straight and in a horizontal position, he or she may be able to do so for 25 - 30 seconds only while one with a normal thyroid sons can maintain such a position for 60 - 120 seconds. Toe standing and step climbing may also bring out muscle weakness that is otherwise not so apparent. In the more extreme forms of muscular involvement, there is not only weakness but also atrophy. Wasting of the temporal and interossei muscles (option D) may be noted in a considerable number of patients, and in a few, wasting of all skeletal muscles. This wasting may go so far as to bear a close resemblance to progressive muscular atrophy. Occasionally, the myopathy may shade into the picture of a polymyositis.

Joint pain such as hip pain is one of the symptoms of and underactive thyroid and is seen in hypothyroidism rather than hyperthyroidism. The cause is often excess fluid accumulation in the joint space and swelling.

How well did you know this?
1
Not at all
2
3
4
5
Perfectly
10
Q

A 30-year-old woman presents to your practice with complaint of episodes of headache, sweating, palpitation, tingling sensation in her hands, shortness of breathو and severe anxiety. A thorough medical workup reveals no pathological finding. During an episode of these symptoms, which one of the following laboratory findings is most likely to be found?

A. Increased free plasma epinephrine.
B. Decreased serum glucose.
C. Decreased serum calcium.
D. Increased serum amylase.
E. Lymphocytosis

A

A. Increased free plasma epinephrine.

Episodic attacks of a constellation of symptoms resembling anxiety are consistent with pheochromocytoma as the diagnosis.

Pheochromocytoma is a rare tumour of adrenal gland tissue that results in increase in the level of adrenal catecholamines. Catecholamines are responsible for sympathetic (fight and flight response) and affect the heart rate, metabolism, and blood pressure. Patients with pheochromocytoma may be asymptomatic or have episodic symptoms of palpitations, flushing, sweating, headache, tremor, and anxiety due to catecholamine excess. Severe hypertension is often seen; however, up to 15% of patients are normotensive. Pheochromocytomas can be associated with familial syndromes such as von Hippel–Lindau disease, multiple endocrine neoplasia type 2 (MEN II), familial paraganglioma syndrome, and neurofibromatosis.

Biochemical findings during an attach are increased serum epinephrine and metanephrine, hyperglycemia, hypercalcemia, and erythrocytosis.

There is currently no consensus as to the optimal biochemical test for pheochromocytoma.

Plasma metanephrine testing has the highest sensitivity (up to 99%) for detecting a pheochromocytoma, but it has a lower specificity (85%). This test is superior to measurement of circulating epinephrine and norepinephrine because plasma metanephrines are elevated continuously, unlike epinephrine and norepinephrine, which are secreted intermittently.

Measurement of 24-hour urinary metanephrines and catecholamines yields a sensitivity of 90% and specificity of 98%. It is often used as a screening test for low-risk patients.

Test selection criteria include the following:
* Use plasma metanephrine testing in patients at high risk (ie, those with predisposing genetic syndromes or a family or personal history of pheochromocytoma)
* Use 24-hour urinary collection for catecholamines and metanephrines in patients at lower risk

NOTE - Thyrotoxicosis, hypoglycemia, anxiety or panic attacks, hyperthyroidism, adrenal medullary hyperplasia, familial dysautonomia, and intracranial lesions may also have similar symptoms. Various tumors including neuroblastomas, ganglioneuroblastomas, and ganglioneuromas may mimic pheochromocytomas.

How well did you know this?
1
Not at all
2
3
4
5
Perfectly
11
Q

A 55-year-old woman presents to your clinic with a swelling in the left lower part of her neck. The swelling has developed over the past 4 months, is painless, and feels rubbery and regular on palpation with no tenderness. She is otherwise healthy and asymptomatic. Ultrasound confirms the presence of a thyroid nodule; therefore, you perform a fine needle aspiration (FNA) sampling and send it for pathology assessment. The pathology result reports only the presence of follicular cells. Which one of the following would be the most appropriate next step in management?

A. CT scan of the neck.
B. Direct laryngoscopy.
C. MRI of the neck.
D. Repeat the FNA.
E. Excisional biopsy of the lump.

A

E. Excisional biopsy of the lump.

Nodular thyroid disease is a common clinical problem. Steps to consider in approaching to a thyroid nodule and excluding malignancy include (1) a thorough clinical evaluation, (2) thyroid function tests (TFT), and (3) an ultrasound exam of the thyroid gland and fine needle aspiration (FNA) biopsy if the nodule is larger than 1 cm.

Thyroid FNA biopsy is the most accurate test for determining malignancy, and is the essential and integral part of evaluation of a thyroid nodule. When guided by ultrasound, there results are even more accurate.

FNA results are classified as diagnostic (satisfactory) and non-diagnostic (unsatisfactory). Unsatisfactory smears, comprising 5-10% of all results, are due to hypocellularity of the specimen, usually caused by cystic fluid in the specimen, bloody smear, or inappropriate preparation.

Diagnostic smears are conventionally classified as:

  • Benign
    Benign cytology comprise 60-70% of all reports, is negative for malignancy, and incudes colloid nodule, cysts, or Hashimoto thyroiditis.
  • Malignant
    Malignant cytology comprise 5% of all reports and is positive for malignancy including primary thyroid tumors or non-thyroid metastatic cancers. Papillary carcinoma is the most common type of thyroid malignancy.
  • Indeterminate (atypical)
    Indeterminate or suspicious specimens, comprising 10-20% of all reports, include atypical changes, Follicular neoplasms, or Hurthle cells. Overall, the indeterminate is associated with a risk of malignancy anywhere between 15% to 60% risk of malignancy (average 30%), depending on the specifics of the report.

Of these conditions, the main difficulty arises in cases of follicular neoplasm because it is impossible to distinguish a follicular adenoma (a benign lesion) from a follicular carcinoma on cytological assessment. Therefore, follicular lesions therefore often require excision and full examination of the lesion and its capsule before a definitive diagnosis can be made.

In general, lesions with atypical features are best completely excised for full evaluation and making a definite diagnosis.

Option D: Repeating the FNA under ultrasound guide is used in cases of non-diagnostic reports in a reattempt to obtain an adequate and appropriately prepared sample.

Option B: Laryngoscopy adds no diagnostic vale to evaluation of a nodule within the thyroid gland and a completely irrelevant option here.

Option A and C: CT scan or MRI are not routinely indicated in the assessment of thyroid nodules because they cannot reliably distinguish benign from malignant lesions. The main indication for CT scan or MRI is to determine the presence and extent of retrosternal extension and the presence and degree of tracheal compression in the presence of obstructive symptoms.

How well did you know this?
1
Not at all
2
3
4
5
Perfectly
12
Q

A 54-year-old woman presents with a swelling in the neck which has progressively enlarged for the past few months. Investigations reveal the mass to be goiter. Which one of the following symptoms, if present, should make you think of urgent surgery?

A. Retrosternal extension.
B. Hoarseness.
C. Dysphagia.
D. Dyspepsia.
E. Puffiness of her face on raising her arms above the shoulder.

A

E. Puffiness of her face on raising her arms above the shoulder.

Goiter refers to an enlarged thyroid gland. Causes of goiter include autoimmune disease, the formation of one or more thyroid nodules and iodine deficiency. Goiter occurs when there is reduced thyroid hormone synthesis secondary to biosynthetic defects and/or iodine deficiency, leading to increased thyroid stimulating hormone (TSH). This stimulates thyroid growth as a compensatory mechanism to overcome the decreased hormone synthesis.

Elevated TSH is also thought to contribute to an enlarged thyroid in the goitrous form of Hashimoto thyroiditis, in combination with fibrosis secondary to the autoimmune process in this condition. In Graves’ disease, the goiter results mainly from stimulation by the TSH receptor antibody.

Substernal goiter may obstruct the thoracic inlet. Pemberton sign refers to faintness with evidence of facial congestion and cyanosis due to external jugular venous obstruction when the arms are raised above the head, a manoeuvre that draws the thyroid into the thoracic inlet.

Of the option, urgent surgery in only indicated who have troublesome compressive symptoms (positive Pemberton sign) and/or fail to respond to medical therapy.

Option A: Retrosternal extension of the thyroid gland per se is not an indication for emergency surgery unless it causes compressive signs and symptoms as explained above.

Option B, C and D: None of these signs or symptoms is an indication for emergency surgery. Dyspepsia is not likely to be a direct mass effect of goiter. Dysphagia or hoarseness might be caused by a direct mass effect but they do not indicate the need for emergency surgery.

Topic Review: Goitre
Causes of goitre:
* Hashimoto thyroiditis
* Graves’ disease
* Familial or sporadic multinodular goiter
* Iodine deficiency
* Follicular adenoma
* Colloid nodule or cyst
* Thyroid cancer
How well did you know this?
1
Not at all
2
3
4
5
Perfectly
13
Q

Ethan is a 16-year-old boy who has presented to your clinic with breast enlargement shown in the following photograph. The breast enlargement was first noticed four months ago as a small swelling under the nipple that has enlarged gradually in the next two months and has remained the same size since then. He has been your patient since the age of nine years with the complaint of asthma that is currently controlled on inhaled salbutamol on an as-needed basis and fluticasone three times daily. He takes no other medications. Physical examination reveals mild tenderness of breasts. The rest of the examination is unremarkable. Which one of the following is the most appropriate management for him?

A. Changing his asthma medication.
B. Reassurance.
C. Mastectomy.
D. Testosterone injection.
E. Ultrasound of the testes.

A

B. Reassurance.

The photograph shows bilateral enlargement of breast with a discoid fashion in a pubertal boy consistent with gynecomastia as the most likely diagnosis. Gynecomastia can be physiologic or nonphysiologic (pathologic).

Physiologic gynecomastia has three phases of occurrence, corresponding to times of hormonal changes. The first peak occurs in the neonatal period due to transplacental passage of estrogen. Gynecomastia of the neonate is seen in 60% to 90% of neonates and almost always resolve spontaneously within the first year of life.

The second peak occurs during puberty when palpable breast tissue and increase in breast size occurs. It can present as early as 10 years, but the peak of onset is between the ages of 13 and 14 years. The incidence declines in late teenage years. By the age of 17, only 10% of boys have persistent gynecomastia.

The last peak of occurrence is in adults between the ages 50 and 80 years.

For a patient presenting with gynecomastia, a thorough history, including drug history and physical exam should be undertaken to exclude pathologic causes of gynecomastia such as medications and drug abuse, hypogonadism, chronic liver or kidney disease, hyperthyroidism, tumors (e.g. testicular, adrenal) and malnutrition. In 25% of cases with gynecomastia no identifiable cause is found (idiopathic gynecomastia).

Ethan is 16 years old and has no remarkable clues in history indicating a pathologic cause. This suggests he has pubertal gynecomastia. Pubertal gynecomastia often resolves between 6 months and 2 years of onset and requires no active management except providing explanation and reassurance.

Option A: Although medications are the cause of gynecomastia in 10 to 25% of cases, salbutamol and fluticasone (and almost none of other asthma medication) has shown a relation with development of gynecomastia, and an advice for a change of asthma medications is not a correct option.

Option C: Mastectomy is considered for patients with persistent gynecomastia who are distressed by their breast appearance due to cosmetic reasons.

Option D: Testosterone injection may be an option for patients with hypogonadism for general treatment of hypogonadism and not merely for the gynecomastia.

Option E: Testicular cancers should be considered in younger adults presenting with pathologic gynecomastia. While some authors advocate routing use of ultrasound for detection a non-palpable testicular mass, the majority advocate routine use of this modality in patients who have abnormal scrotal exam findings such as a testicular mass, signs of feminization, and other indicators of testicular malignancies. Ethan has the provisional diagnosis of physiologic gynecomastia and there is no clues in the history and physical examination to mandate a scrotal ultrasound.

How well did you know this?
1
Not at all
2
3
4
5
Perfectly
14
Q

Adrian is a 17-year-old boy, who has presented to your GP clinic with complaint of painful breast enlargement for the past three months as shown in the following photograph. He has asthma that is well-controlled on inhaled salbutamol and beclomethasone. He has no other medical condition, otherwise takes no other medications, and enjoys a good health. He plays football in his school team. He denies using illicit drugs. Physical examination shows unilateral right breasts enlargement starting from the nipple and spreading concentrically. Touching the breast causes mild pain. There is no discharge or lymphadenopathy. The rest of the examination including testicular examination is unremarkable. Which one of the following is the most likely cause of this presentation?

A. Inhaled salbutamol.
B. Inhaled beclomethasone.
C. Misuse of anabolic steroids.
D. Physiologic gynecomastia of adolescents.
E. A testicular tumor.

A

C. Misuse of anabolic steroids.

The photograph and the description of physical findings in in favor of gynecomastia as the most likely diagnosis.

The three major causes of gynecomastia are physiologic, idiopathic and drug/substance abuse related. Physiologic gynecomastia comprise 25% of all cases and is seen in neonatal, pubertal, and in males between 50 to 80 years of age. Medications including drugs of abuse such as alcohol, anabolic steroids, amphetamine and opiates account for 10 to 25% of cases. In 25% of cases gynecomastia is idiopathic and no identifiable cause is found. Other less likely causes are primary and secondary hypogonadism, chronic liver disease, chronic renal disease, tumors (e.g., testicular, adrenal), malnutrition/ starvation, and hyperthyroidism as well as other rarer causes.

In this boy, persistent pubertal gynecomastia (not an option) can be the best option, given the absence of other physical findings that may suggest a medical condition such as hypogonadism or testicular tumors. Persistent pubertal is classified under idiopathic gynecomastia, and defined as pubertal gynecomastia not resolving after 2 years, or persisting beyond 17 years of age.

NOTE - by the age of 17, pubertal gynecomastia is resolved in 90% of boys. Gynecomastia beyond this age is no more termed physiologic and is categorized under **idiopathic pathologic gynecomastia**.

In the absence of idiopathic gynecomastia that statistically seems the likely explanation, medications and specific substances should be considered as a possibility. Many medications, based either on anecdotal reports or studies of different quality, are known to cause gynecomastia. Of Adrian’s medications, neither inhaled salbutamol (option A) nor inhaled betamethasone (option B) can cause gynecomastia. Another possibility to consider, especially in younger patients with athletic behavior, is misuse of anabolic steroids. This boy is a football player and is likely to have used or is using anabolic steroids for enhanced performance. In younger patients with gynecomastia who also have athletic behaviors, misuse of anabolic steroids should always be considered as a possibility, an in fact the best option among others in this scenario.

Option D: Physiologic gynecomastia of adolescents is expected to resolve between 6 months to 2 years of onset and usually does not persist beyond 17 years of age. On the other hand, the breast enlargement in the photograph exceeds 5cm in diameter (macromastia). Macromastia suggest pathologic rather than physiologic gynecomastia.

Option E: Testicular tumors are an important but not common cause of gynecomastia, especially in younger patients. Although less likely, testicular tumors should be considered a possibility and investigated thoroughly, especially if there is a testicular lump. However, in the absence of signs and symptoms, a testicular tumor is less likely to be the cause of such presentation.

How well did you know this?
1
Not at all
2
3
4
5
Perfectly
15
Q

A 68-year-old man has bilateral breast enlargement as shown in the accompanying photograph. His medical history includes hypertension, rheumatoid arthritis (RA), and gastroesophageal reflux disease (GERD). He takes nifedipine and hydrochlorothiazide for hypertension, ibuprofen for the RA, and ranitidine for the reflux. He is a heavy alcohol drinker and takes 8-12 standard units of alcohol every day. He also smokes two packs of cigarettes per day. On examination, he has a blood pressure of 145/98 mmHg, pulse rate of 92 bpm and respiratory rate of 18 breaths per minute. There is bilateral breast enlargement consistent with gynecomastia. In addition, he has bilateral parotid enlargement. The abdomen is soft and non-tender with no ascites or organomegaly. Which one of the following is most likely to have caused this presentation?

A. Smoking.
B. Ibuprofen.
C. Ranitidine.
D. Nifedipine.
E. Alcoholic liver disease.

A

E. Alcoholic liver disease.

The photograph and the physical findings are highly suggestive of gynecomastia. Gynecomastia, is the development of female-like breasts in a male caused by proliferation of breast glandular tissue.

Gynecomastia can be physiologic in neonates, adolescents and older males between the age of 50 to 80 years, or pathologic (nonphysiologic) and caused by a medical condition, medications, or substances of abuse (alcohol, marijuana, heroin, anabolic steroids). In idiopathic gynecomastia, no identifiable cause is found. The three major etiologies of gynecomastia are physiologic (25% of all cases), idiopathic (25% of all cases), and medications/ substance use (10-25%). Other contributing factors include chronic liver disease, malnutrition, primary or secondary hypogonadism, tumors (e.g. testicular, adrenal), hyperthyroidism, chronic kidney disease/ dialysis, and other rarer causes.

It is important to differentiate gynecomastia from pseudogynecomasita that is fat deposition in obese men and also from breast cancer, although male breast cancer is rare and comprise only 1% of all breast cancers. Nonetheless, it should always be considered as possibility in men presenting with gynecomastia, especially if it is unilateral and painless.

Medications are another common etiology for breast cancer. The most commonly reported medications with a high association with gynecomastia are inhibitors of testosterone production (e.g. cyproterone acetate, spironolactone, finasteride, flutamide, ketoconazole, etc.), anabolic steroids, cimetidine, digoxin (digitalis), captopril, and methyldopa.

The following table lists medications that have been reported to be associated with gynecomastia. Most of the listed the listed drugs are based on anecdotal reports and do not have high quality evidence supporting a cause-and-effect relationship. Using evidence-based criteria, there is good evidence of a true association with estrogens, spironolactone, cimetidine, ketoconazole, growth hormone, gonadotropins, antiandrogens therapies and 5-alpha reductase inhibitors.

Medications with fair evidence include first-generation and atypical antipsychotics in adults, calcium channel blockers, omeprazole, HIV drugs, alkylating agents, anabolic steroids, alcohol and opioids. Other drugs only have been reported anecdotally without quality supporting evidence.

This patient is on multiple medications and also taking excess alcohol. Of his medications, nifedipine (option D) can be the culprit for the gynecomastia (fair evidence). Ranitidine (option C) could be a less likely, yet not impossible cause of this presentation. Ibuprofen (option B) has not shown any associated with gynecomastia, neither does smoking (option A).

In this patient, the gynecomastia and bilateral parotid enlargement can be early indicators of alcoholic liver disease and the best option among others. Later in the course of the disease, other signs such as ascites, jaundice, pruritus, etc. can develop. It should be noted though that alcohol alone (not an option) even in the absence of liver disease can produce such clinical picture. Alcoholic people can develop gynecomastia due to phytoestrogen and parotid enlargement through disrupted fat metabolism and fat deposition in parotid glands.

NOTE - given the absence of other features of liver disease in the question, and alcohol in the options, it is only a matter of justifying the best option among of the others. The only two options that fit the scenario best are alcoholic liver disease and nifedipine. Since the patient has been on long-term nifedipine but just recently has developed gynecomastia, it is less likely (but not impossible) that nifedipine has caused gynecomastia compared to liver disease for which an additional fair clue (parotid enlargement) is provided in the scenario.

How well did you know this?
1
Not at all
2
3
4
5
Perfectly
16
Q

Benjamin is 65 years old and a regular patient in your GP clinic. He has presented with complaint of left breast enlargement for the past three months. He has congestive heart failure (CHF) diagnosed two years ago, for which he was started on metoprolol, captopril, and digoxin. Four months ago, he developed ankle edema, nocturnal cough, and exertional dyspnea. Based on the diagnosis of CHF decompensation, spironolactone was added to his medications with adequate control of his symptoms. He does not smoke, nor does he drink alcohol. His family history is significant for breast cancer in his mother at the age of 58 years and in his sister at the age of 50 years. On examination, a tender round rubbery mass 4x3 cm in size and concentric with the left nipple is palpated. The mass is regular and mobile. The other breast is normal. There is no lymphadenopathy, nipple discharge or skin changes. Which one of the following would be most appropriate next step in management?

A. Ultrasound of the breast.
B. Cessation of spironolactone.
C. Fine needle aspiration (FNA) of the breast lump.
D. Cessation of digoxin.
E. Excision of the breast lump.

A

A. Ultrasound of the breast.

The clinical findings in in Benjamin is highly suggestive of gynecomastia as the most likely diagnosis.

Gynecomastia is characterized by proliferation of breast glandular and adipose tissue causing a female -like breast in a male. Gynecomastia can be physiologic in neonates, pubertal, and in adult males aged 50-80 years. Physiologic gynecomastia accounts for 25% of all cases of gynecomastia. Pathologic (nonphysiologic) causes of gynecomastia include medications and substance abuse (10-25%), liver disease or cirrhosis (8%), primary/secondary hypogonadism (10%), tumors (3%), hyperthyroidism (1.5-2%), renal disease/ dialysis (1%), and other identifiable causes (6%). In 25% of cases, including persisting pubertal gynecomastia (gynecomastia not resolving within 6 months to 2 years, or persisting after 17 years of age), no identifiable cause is found (idiopathic gynecomastia).

Gynecomastia is not palpable unless the diameter of the glandular tissue exceeds 0.5 cm. Gynecomastia has four characteristic features on physical examination:
1. centrally located
2. symmetrical in shape
3. usually bilateral
4. tender to palpation

On physical examination the clinical findings include palpation of a rubbery or firm mass concentric with the nipple and areola. Tenderness and discomfort are often evident on palpation. It is bilateral in over 90% of patients, but sometimes patients present with unilateral enlargement or bilateral enlargement but one side large than the other. One side may enlarge weeks to months later before the other.

Although, unilaterality alone is not an indicator of pathologic gynecomastia, some studies suggest that it could be more common with specific etiologies such as medications.

Gynecomastia with a diameter exceeding 5 cm is referred to as macromastia. Macromastia in more cases has a pathologic rather than physiologic etiology, especially if unilateral.

For patients presenting with gynecomastia, a thorough history and physical examination should be undertaken. In history, important points to underpin are pubertal development and stage (in younger adults), any medications taken or substances used by the patient, medical conditions such as thyroid, liver or kidney disease, libido, and history of cancer. In physical examination, differentiation between true gynecomastia and other conditions such as pseudogynecomastia and, most importantly, breast cancer is very important. Other clues to look for are signs of liver disease (jaundice, pruritus, ascites, enlarged parotid glands, spider nevi, etc.), renal disease, hypogonadism (feminization, decreased libido and sexual function), hyperthyroidism, testicular cancer (e.g., testicular mass), and drug abuse (opiates, alcohol, anabolic steroids).

If based on the history and physical findings, the case turns out to be physiologic gynecomastia, no further evaluation is required. Similarly, asymptomatic and pubertal gynecomastia do not require further tests and should be reevaluated in six months. On the other hand, further evaluation is indicated in the following situations:
* Macromastia (breast size exceeding 5 cm in diameter)
* A lump that is tender, of recent onset, progressive, or of unknown duration
* Signs of breast malignancy i.e. stony hard mass, irregular borders, being fixed to the surrounding tissues, lymphadenopathy, unintentional weight loss, loss of appetite, skin changes such as dimpling, or nipple discharge.
* There is concerns about testicular cancer or other non-breast malignancies

A serum chemistry panel may be helpful for renal or liver disease is suspected. Free or total testosterone, luteinizing hormone (LH), estradiol, and dehydroepiandrosterone sulfate levels are used to evaluate a patient with possible feminization syndrome. Thyroid-stimulating hormone (TSH) and free thyroxine levels should be obtained if hyperthyroidism is suspected.

Routine use of sonography for patients with nonphysiologic gynecomastia has been a place of debate; however, recent studies as well as expert opinion suggest that ultrasonography as a readily available, non-invasive and non-expensive investigation be used to establish a diagnosis of true gynecomastia and its size and also exclusion of other differential diagnoses such as cancer, pseudogynecomastia, lipoma, hematoma, etc., especially if findings are suggestive.

Also, scrotal sonography for possible testicular cancer is endorsed and recommended for the those with a testicular mass found on physical examination, increased serum estradiol or HCG levels, macromastia, and otherwise unexplained gynecomastia, especially in younger patients.

Benjamin’s breast enlargement is very likely to have been caused by spironolactone. There is strong evidence for spironolactone as an inducer of gynecomastia mostly through inhibition on testosterone production, but cessation of spironolactone (option B) on which his CHF is stable is not a wise advice as it can cause deterioration of his CHF symptoms. Digoxin has been reported as a cause of gynecomastia, but compared to spironolactone, the evidence is fair. Nonetheless, it can still be the cause; though less likely than spironolactone. Even though, cessation of digoxin (option D) can result in CHF decompensation, and should not be advised.

Of the option, a breast ultrasound is a reasonable choice because it can safely exclude other causes of breast enlargement, confirm a certain diagnosis, and provide relief in case it confirms the provisional diagnosis of gynecomastia.

FNA of the breast lump (option C) must be considered if imaging suggests malignancy. Excision of the breast lump (option E) for now is unnecessary because it is invasive and adds no diagnostic benefit.

How well did you know this?
1
Not at all
2
3
4
5
Perfectly
17
Q

A 62- year-old woman presents with bloody discharge from her left nipple. The family history is significant for a cousin recently diagnosed with breast cancer. On examination, no mass is palpated. Which one of the following is the most likely diagnosis?

A. Breast duct papilloma.
B. Breast duct ectasia.
C. Paget disease of the breast.
D. Mastitis.
E. Breast duct carcinoma.

A

A. Breast duct papilloma.

Nipple discharge, as a sign, may have a variety of underlying causes. Some of these causes are as follows:

  • Bloody discharge – bloody discharge from nipple is usually caused by benign ductal papilloma, but breast neoplasms including breast duct carcinoma and Paget disease of the breast should always be excluded.
  • Serous, green, yellow-brown discharge – it is usually due to benign fibrocystic changes
  • Toothpaste (worm)-like discharge – it is characteristic of mammary ductal ectasia.

The bloody nipple discharge of this woman is most likely to have been caused by benign ductal papilloma; however, breast duct carcinoma (option E) and Paget disease of the breast (option C), as less common possibilities, should be considered as well.

Option B: Breast duct ectasia presents with worm-like or toothpaste-like but not bloody nipple discharge.

Option D: Mastitis presents with local tenderness and redness of the breast. The clinical picture is completely different from this case scenario. Moreover, it is a very uncommon condition in a woman of this age.

How well did you know this?
1
Not at all
2
3
4
5
Perfectly
18
Q

Lucas is 67-year-old patient of yours in your GP clinic. He has congestive heart failure (CHF) diagnosed three years ago for which he has been on carvedilol, perindopril, and digoxin with improvement in symptoms. Six months ago, due to decompensation of his CHF, evident by increased dyspnea and ankle edema, and after specialist advice, he was started on spironolactone. Today, he has presented with complaint of painful unilateral breast enlargement. He does not smoke, and drinks only on social occasions and within safe limits. His past medical history, besides the CHF, is myocardial infarction seven years ago. His family history is remarkable for breast cancer in his mother, of which she died 15 years ago, and in his sister at the age of 52. On examination, you notice a left breast lump concentric with the nipple which is firm and rubbery and slightly tender. There is no nipple discharge, ulceration or regional lymphadenopathy. The other breast is normal. Which one of the following could be the most likely cause of this presentation?

A. Breast cancer.
B. Digoxin.
C. Spironolactone.
D. Carvedilol.
E. Perindopril.

A

C. Spironolactone.

The description of the breast enlargement in Lucas is consistent with gynecomastia as the most likely diagnosis. Gynecomastia is defined histologically as proliferation of breast glandular and adipose tissue, and clinically as the development female -like breast in a male. Gynecomastia, in most cases is due to increased ratio of female sex hormones to testosterone, and presents with a firm rubbery mass concentric with the nipple. It is bilateral in most cases but in 5-10% of men it can be unilateral and for unknown reasons more often on the left side.

Gynecomastia can be physiologic in neonates, adolescents and older males between the age of 50 to 80 years, or pathologic (nonphysiologic) and caused by a medical condition, medications, or substances of abuse (alcohol, marijuana, heroin, anabolic steroids). In idiopathic gynecomastia, no identifiable cause is found. The three major etiologies of gynecomastia are physiologic (25% of all cases), idiopathic (25% of all cases), and medications/ substance use (10-25%).

Other contributing factors include chronic liver disease, malnutrition, primary or secondary hypogonadism, tumors (e.g. testicular, adrenal), hyperthyroidism, chronic kidney disease/ dialysis, and other rarer causes.

It is important to differentiate gynecomastia from pseudogynecomasita that is fat deposition in obese men and also from breast cancer. Breast cancer is rare in men and comprise less than 1% of all cases of breast cancer. On the other hand, breast cancer (option A) presents differently with a stony hard lump that is usually immobile and attached to the surrounding structures with possible skin dimpling, lymphadenopathy and nipple discharge, or ulceration in advanced disease. Nonetheless, it should always be considered as possibility in men presenting with gynecomastia, especially if it is unilateral and painless.

Medications are another common etiology for breast cancer. The most commonly reported medications with a high association with gynecomastia are inhibitors of testosterone production (e.g. cyproterone acetate, spironolactone, finasteride, flutamide, ketoconazole, etc.), anabolic steroids, cimetidine, digoxin (digitalis), captopril, and methyldopa.

The following table lists medications that have been reported to be associated with gynecomastia. Most of the listed the listed drugs are based on anecdotal reports and do not have high quality evidence supporting a cause-and-effect relationship. Using evidence-based criteria, there is good evidence of a true association with estrogens, spironolactone, cimetidine, ketoconazole, growth hormone, gonadotropins, antiandrogens therapies and 5-alpha reductase inhibitors.

Medications with fair evidence include first-generation and atypical antipsychotics in adults, calcium channel blockers, omeprazole, HIV drugs, alkylating agents, anabolic steroids, alcohol and opioids. Other drugs only have been reported anecdotally without quality supporting evidence.

Of the medications Lucas is taking, spironolactone, digoxin , and perindopril (ACE inhibitor) have been reported to have a potential to cause gynecomastia. Spironolactone has the best evidence to cause gynecomastia, and most likely to have caused this presentation. Although gynecomastia can occur even after prolonged use of a potential medication, the fact that Lucas has developed gynecomastia after being started on spironolactone, is in line with this option as well.

Carvedilol (beta blocker) has not shown a meaningful association with gynecomastia.

How well did you know this?
1
Not at all
2
3
4
5
Perfectly
19
Q

Mia, 45 years of age, presents for advice about reducing the risk of breast cancer after her mother was diagnosed with breast cancer at the age of 60 years. She started menstruation at the age of 17 and has three children, the first born when she was 37 years. She has been on combined oral contraceptives for the past four years and is happy with that. She smokes 10 to 15 cigarettes a day. Her past medical history incudes benign breast disease including simple breast cyst and fibrocystic changes of the breast. Of the factors in the history, which one could not be a risk factor for development of breast cancer in her?

A. The history of previous breast diseases.
B. Smoking.
C. Family history of breast cancer in her mother.
D. Combined oral contraceptives.
E. Her first pregnancy at the age of 37.

A

A. The history of previous breast diseases.

Of the given option, only Mia’s history of previous breast diseases cannot be a cause for breast cancer. All other options, with different relative risks, has been shown to be associated with development of breast cancer as risk factors.

Some breast diseases have a relation with breast cancer and are associated with a slight increase in the risk of a woman catching breast cancer while others not. Benign breast diseases represent a spectrum of breast disorders noticed incidentally during imaging, because of the symptoms or presence of a breast lump. Some of such diseases are associated with an increase in the patient’s future risk of developing breast cancer. These diseases are rather risk markers than premalignant, because those cancers that subsequently develop are not necessarily in the area the breast disease exists and the malignancy may occur in the contralateral breast.

Benign breast disease can be categorized histologically into three categories:
1. Nonproliferative
These diseases are not generally associated with an increased risk of breast cancer and include: simple cyst (the most common non-proliferative breast disease), papillary apocrine change, epithelial related calcification, and mild hyperplasia of usual type. It is important to note that terms such as fibrocystic changes, fibrocystic disease, chronic cystic mastitis, and mammary dysplasia refer to nonproliferative lesions. These terms are not clinically useful because they encompass a heterogeneous group of diagnoses, not a single diagnosis.

2. Proliferative without atypia
Proliferative lesions without atypia include usual ductal hyperplasia, intraductal papillomas, sclerosing adenosis, and radial scars. These lesions are associated with a small increased risk of developing breast cancer (approximately 1.5-2 times that of the general population). Fibroadenomas are associated with increased risk of breast cancer only if the fibroadenoma is complex, if there is adjacent proliferative disease, or if there is a family history of breast cancer. For the majority of women with fibroadenomas the cancer risk is not increased.

3. Proliferative with atypical hyperplasia
Proliferative breast diseases with atypical hyperplasia includes atypical ductal hyperplasia and atypical lobular hyperplasia. Atypical hyperplasia is pathologic diagnosis, usually found as an incidental on biopsy of mammographic abnormalities or breast lumps. These lesions have some, but not all, features of ductal carcinoma in situ or lobular carcinoma in situ. These lesions, especially if multifocal are associated with a significantly increased risk of breast cancer.

Mia’s breast cyst and the fibrocystic disease are both nonproliferative lesions from category one and does not confer an increased risk of breast cancer.

Option C: Most women with breast cancer do not have a family history; however, having one or more first-degree relatives or second-degree relatives on the same side of family with breast cancer is associated with increased risk of breast cancer. Mia, with one first-degree relative (mother) diagnosed with breast cancer after the age of 50, is at slightly increased risk of breast cancer (1.8 times the normal population). Despite being modest, family history can play a role as cause of breast cancer.

Option B: Many studies have shown that tobacco smoking may be associated with an increased risk of breast cancer. While there are some limitations to the evidence, studies have been generally consistent in supporting a link between tobacco smoking and increased risk of breast, particularly in women who have started smoking when they were younger or have started smoking many years before having their first child.

Option D: Combined’ oral contraceptives (COCs) are associated with a small increase in risk of breast cancer while being taken. This risk is estimated to be 7% for every 5 year use and goes down again when stopped. It is estimated that 0.7% of breast cancers each year in Australia are attributable to the use of COCs.

Option E: First full-term pregnancy after the age of 35 is associated with increased risk of breast cancer (RR: 2.1-4).

20
Q

Barney, 67 years old, presents with a left breast mass he noticed felt a few days ago. On physical examination, there is a non-tender left breast mass which is firm and non-mobile. There is no lymphadenopathy. An ultrasound scan is arranged, the result of which reports a 2x3 cm hypoechoic mass with irregular borders. Which one of the following is the most appropriate next step in management?

A. Excisional biopsy.
B. Mastectomy.
C. Fine needle aspiration (FNA) biopsy.
D. Follow-up ultrasound in 6 months.
E. Follow-up mammography in 6 months.

A

C. Fine needle aspiration (FNA) biopsy.

The approach to a breast lump in male patients is almost the same as for females and include physical examination, imaging and (if indicated) biopsy and histological studies.

Clinical findings suggestive of a malignant lesion are the stony-hard consistency of the lesion, irregular borders, being non-tender, immobility, and skin changes (e.g. dimpling). Ulceration of the skin is seen in advanced disease. Lymphadenopathy could be a feature but it could be absent in early disease.

In approach to a suspicious mass, imaging should follow the physical examination. Mammography is the standard modality; however, it can be associate with technical difficulties due to small size of breasts in men. Ultrasound can reliably be used for such purpose. On ultrasound, a malignant lesion has characteristic features including:
* Hypoechogenicity
* Irregular and ill-defined borders
* Spiculated margins
* Being taller than broader
* Posterior acoustic shadowing
* Microcalcifications

Any suspicious mass should be biopsied to confirm the diagnosis and to assay for estrogen and/or progesterone receptors as well as human epidermal growth factor receptor 2 status.

Barney’s ultrasound study shows hypoechogenicity and irregular borders of the mass suggestive of malignancy. In addition, firmness of the mass and its immobility are in favor of malignancy as well. Therefore, the next step to consider is sampling via FNA or core needle biopsy.

FNA biopsy is convenient and minimally invasive and can provide adequate tissue for diagnostic purposes in many cases; however, in up to 25% of patients, the sample is insufficient for diagnosis. Compared to FNA, core needle biopsy offers a more definitive histological diagnosis, avoids inadequate sampling, and usually distinguishes between invasive versus in situ cancer.

Open biopsy (excisional biopsy) may be an option if less invasive measures such as FNA or core needle biopsy fail to obtain adequate samples.

While the mass has suspicious features of malignant lesion, follow-ultrasound (option D) or mammography (option E) are inappropriate, and action should be taken for a definite diagnosis and management. Mastectomy (option B) would be the likely surgical component of the treatment if biopsy establishes a definite diagnosis of breast cancer.

21
Q

All of the following options are features of diabetic autonomic neuropathy EXCEPT:

A. Bradycardia.
B. Urinary retention.
C. Sexual impotence.
D. Diarrhea.
E. Foot ulcers.

A

E. Foot ulcers.

Autonomic neuropathy is one of the complications of diabetes mellitus. Diabetic autonomic neuropathy is common and often a debilitating condition. Clinical manifestations of diabetic autonomic neuropathy include the following:

  • Skin - dry skin (due to lack of sweating), or excessive sweating in defined areas
  • Eye - poor dark adaptation, sensitivity to bright lights
  • Cardiovascular - postural lightheadedness, fainting, orthostatic tachycardia, orthostatic bradycardia and orthostatic hypotension
  • Urinary - urgency, incontinence, dribbling, and bladder emptying abnormalities
  • Gastrointestinal - diarrhea, constipation, nausea and vomiting
  • Sexual - erectile dysfunction and ejaculatory failure in men, loss of ability to reach sexual climax in women

Although foot ulcers, a major source of morbidity in patients with diabetes, are a feature of diabetic neuropathy, it is unrelated to autonomic dysfunction. Diabetic foot ulcers have a different pathophysiology, mostly including peripheral neuropathy and ischemia from peripheral vascular disease. Poor resistance against infections is also a contributing factor.

22
Q

A 29-year-old woman, who has been recently diagnosed with nodular thyroid disease, is worried about developing thyroid cancer. Which one of the following does not increase the risk of thyroid cancer?

A. Family history of benign thyroid disease.
B. Family history of thyroid cancer.
C. Graves’ disease.
D. Chronic goiter.
E. Familial adenomatous polyposis.

A

A. Family history of benign thyroid disease.

While personal history of a benign thyroid diseases, such as thyroid nodules, is associated with increased risk of thyroid cancer, such relationship has not been established for family history of benign thyroid diseases.

Option B: A family history of thyroid cancer in a first-degree relative is a proven risk factor for thyroid cancer. The genetic basis is still undetermined.

Option C: Some research have concluded that Graves’ disease can be associated with increased risk of thyroid cancer. It is not known whether Garves’ disease by itself or the treatments used (radiotherapy or thyroid ablation) increases such risk.

Option D: Chronic iodine deficiency is a risk factor for development of thyroid cancer. Chronic goiter, as a presentation of such deficiency, can increase the risk of thyroid cancer.

Option E: Familial adenomatous polyposis (FAP) is a well-known risk factor for thyroid cancer. FAP is associated with colonic polyposis and a very high risk of colon cancer. There is also an increased risk for other cancers such as papillary thyroid cancer. Gardner syndrome, a subtype of FAP, is also associated with such risk as well.

23
Q

A 32-year-old woman presents to your GP clinic for advice on breast cancer after her maternal aunt was diagnosed with breast cancer
three months ago. She wants to know about the risk factors of breast cancer. She is obese and has a body mass index (BMI) of 45. She smokes 20 cigarettes a day and is on oral contraceptive pills (OCP). The rest of examination, including breast exam, is unremarkable. Which one of the following pieces of advice is the best to reduce the risk of breast cancer in her?

A. Smoking cessation.
B. Weight loss.
C. Regular exercise.
D. Using low-estrogen dose OCPs.
E. Using non-hormonal methods for contraception.

A

B. Weight loss.

It has been estimated that approximately 20%of all cancers are caused by excess weight. The Million Women Study, the largest study of its kind on women has shown that approximately half of cancers can be attributed to obesity in postmenopausal women. The International Agency for Research into Cancer (IARC) and the World Cancer Research Fund (WCRF) reports show that common cancers in obese people are predominantly endometrial, esophageal adenocarcinoma, colorectal, postmenopausal breast cancer, prostate and renal cancers. Less common malignancies associated with obesity are melanoma, thyroid cancer, leukemia, non-Hodgkin’s lymphoma, and multiple myeloma.

The role of obesity in development of breast cancer in women is hypothesized to be due to increased production of endogenous estrogen in the fat tissue and excess exposure of the breast tissue to unopposed endogenous estrogen. For this patient, weight reduction is the most appropriate advice both for general health and risk reduction for breast cancer.

Option A: Smoking have a positive link to a number of malignancies. Some reports advocate tobacco smoking as a risk factor for the development of breast cancer; however, this risk is not as significant as obesity.

Option C: Regular exercise is associated with improved overall health as well as decreased incidence of malignancies. The effect does not seem to be superior to weight reduction for breast cancer.

Option D and E: Use of contraceptive pill has been associated with only a slightly increased risk of breast cancer after 5 years of use. Changing the contraception method or reducing the dose of estrogen is not likely to significantly decrease such risk.

Topic Review - ADDENDUM:
Factors that reduce risk of breast cancer (RR < 1) include the following:

  • Asian, Hispanic, or Pacific islander race
  • Breastfeeding
  • Age < 20 at first pregnancy
  • Tamoxifen use
  • Prior risk-reduction breast surgery
  • History of cervical cancer
  • History of oophorectomy
  • Exercise/active lifestyle
  • Low bone mineral density

Factors associated with RR 1.1-2.0 for breast cancer are as follows :

  • Never breastfed a child
  • Nulliparity (no full-term pregnancies)
  • Late menopause (age >55)
  • Type II diabetes mellitus
  • Obesity (post-menopausal)
  • Personal history of uterine, ovarian, or colon cancer
  • Recent and long-term use of hormone replacement therapy (HRT) containing estrogen and progestin
  • Recent oral contraceptive use
  • Occupation: night shift
  • Tobacco abuse
  • Sedentary lifestyle
  • Inferior cardiovascular health
  • High bone mineral density
24
Q

A 22-year-old man is brought to the emergency department after he collapsed during surfing. On examination, he is tanned, has a blood pressure of 80/60 mmHg and a pulse rate of 100 bpm. You draw blood and start him on intravenous fluids. A bolus of 500 ml normal saline is given, and since there is no response, you give him another 2 liters of normal saline in 2 hours; however, his blood pressure remains low. His laboratory results are back and are remarkable for a serum sodium level of 124 mmol/L and potassium of 6 mmol/L (3.5-5.5). Which one of the following is the most appropriate next step in management?

A. Give him more fluids.
B. Transfuse packed cells.
C. Give him corticosteroids.
D. Start him on dopamine.
E. Give him a bolus of atropine.

A

C. Give him corticosteroids.

The scenario describes a patient with persistent hypotension despite adequate volume replenishment and without any evident source of fluid or blood loss. However, in the light of the laboratory results, another possibility finds its way into the differential diagnoses. This patient has hyponatremia and hyperkalemia making adrenal insufficiency a likely possibility diagnosis. Aldosterone, secreted by adrenal glands, maintains intravascular volume via reabsorption of sodium and water from renal tubules while increasing potassium and hydrogen ions excretion at the same time. Aldosterone insufficiency results in sodium and water loss and hyperkalemia. Accumulation of hydrogen ions gives rise to metabolic acidosis.

This patient has also hyperpigmentation than can either be due to tanning caused by surfing in the sun or adrenal insufficiency. If the latter is the case, hyperpigmentation is a support for the diagnosis, and also indicates that the adrenal insufficiency is primary (Addison disease). Hyperpigmentation is a finding in nearly all patients with primary adrenal insufficiency, and is the most characteristic physical finding. Physiologic response to cortical adrenal insufficiency causes an increase in production of ACTH with melanocyte stimulating hormone (MSH) as a co-product. The elevated MSH results in increased melanin synthesis and causes hyperpigmentation.

The resulting brown hyperpigmentation is generalized, but is most prominent in areas exposed to light (such as the face, neck, and backs of hands) and areas exposed to chronic friction or pressure (such as the elbows, knees, spine, knuckles, waist, and shoulders (brassiere straps). Pigmentation is also prominent in the palmar creases, where it escapes being worn away by friction, and in areas that are normally pigmented, such as the areolae, axillae, perineum, and umbilicus.

Management of adrenal crisis is with parenteral administration of corticosteroids. Hydrocortisone is the drug of choice for patients with known diagnosis of adrenal insufficiency, while dexamethaosne is the preferred medication for those without a previous diagnosis of adrenal insufficiency, because it is not measured in serum cortisol assays.

(Option A and B) In patients with adrenal insufficiency, fluid replacement does not increase the blood pressure. Corticosteroids alone will treat the hypotension without any further need for fluid replacement.

(Option D) Vasopressors such as dopamine or epinephrine are not useful for treatment of adrenal crisis because the vascular tone is already normal. Vasoconstriction will not replenish the deficient volume.

(Option E) Atropine is an anticholinergic agent that counteracts parasympathetic effects such as bradycarida. Atropine will not treat hypotension caused by a non-cardiovascular pathophysiology.

25
Q

A 40-year-old woman presents to you clinic with a painless lump in her right breast that she noticed three days ago. Which one of the following is the next best step in management?

A. Fine needle aspiration (FNA).
B. Mammography.
C. Ultrasonography.
D. Core biopsy of the lump.
E. Complete excision of the lump.

A

B. Mammography.

In approach to a palpable mass, mammography is the initial diagnostic test to exclude occult abnormalities in the noninvolved tissue, and to rule out carcinoma. If the patient is 30 years old or younger, ultrasonography (option C) would be the initial diagnostic modality, because the dense breast tissue in this age group makes it difficult for mammogram to detect small lesions and can leads to false negative results.

Simple FNA (option A) is usually performed after mammography. If FNA is done before mammography or ultrasound, a 2-week interval should be waited because small hematomas from the needle aspiration may cause false-positive results.

Although an ultrasound can distinguish a cyst from a solid mass, FNA is the preferred method when a mass is palpable because the procedure is both diagnostic and therapeutic. FNA frequently establishes the etiology of a cyst, provides pain relief in a symptomatic cyst, and results in a clinical breast exam free of interfering masses. This method also allows easy diagnosis in the office setting, which, in some cases, relieves anxiety for the patient.

Option E: A complete excision of the lump is indicated if a malignant breast lesion is highly suspected.

26
Q

A 55-year-old diabetic woman presents with a foot ulcer shown in the following photograph. On examination, she looks ill and has a fever of 38.6°C. Which one of the following is the most appropriate next step in management after wound debridment and taking swabs?

A. Oral amoxicillin – clavulanate.
B. Intravenous ticarcillin - clavulanate.
C. MRI of the foot.
D. Daily wet dressing.
E. Daily wound toilet with povidone iodine.

A

B. Intravenous ticarcillin - clavulanate.

The photograph shows an ulcer on the fifth toe of the right foot with marked inflammation and cellulitis of the surrounding tissue. Moreover, the patient has fever and tachycardia (systemic toxicity). In such cases, after debridement and taking swabs for cultures (!), antibiotics are of significance importance as the next best step in management. The antibiotics of choice for patients with severe infection evident by systemic toxicity is piperacillin + tazobactam or ticarcillin + clavulanate.

The following table outlines choice of antibiotics for different types of diabetic foot ulcers.

Option A: Oral amoxicillin-clavulanate is used for mild infections or ulcers without infection but odorous discharge.

Option C: MRI is indicated if osteomyelitis, as a complication of the ulcer, is suspected. It is not a priority at this stage. Even if was indicated, it would not be a priority compared to debridement, taking swabs, and starting antibiotics.

Option D: Daily wet dressing is used as adjunct therapy to antibiotics for wound care.

Option E: Cytotoxic antiseptics such as povidone iodine delay wound healing and should not be used.

27
Q

A 60-year-old diabetic woman, whose diabetes is well controlled on insulin, comes to your clinic with an ulcer on the dorsum of her right fifth toe which she noted 5 days ago. On examination, a purulent odorless discharge is noted. There is marked inflammation of the fifth toe, extending to the lateral aspect of the foot and ankle. She has a blood pressure of 110/85 mmHg, heart rate of 104 bpm, and temperautre of 38.7 C. Which one of the following is the most appropriate next step in management?

A. Intravenous ticarcillin + clavulanate.
B. Analgesics.
C. X-ray of the foot.
D. MRI of the foot.
E. Debridment of the ulcer.

A

E. Debridment of the ulcer.

In approach to diabetic-related foot ulcers (DRFU), the most important initial step in management is debridement of the ulcer. Debridement allows comprehensive evaluation of the wound, wound bed, and actual wound size. Debridement can convert a chronic wound to an acute one with accelerated healing. It also removes local pressure off the wound.

After the wound is debrided, swabs are taken for bacteriological assessment and culture. This should be performed before antibiotics are started. In the next step, antibiotics should be stared if indicated.

(Option A) The decision as to empirical antibiotic treatment is guided by the characteristics of the ulcer. The following table summarizes the types of diabetic ulcers, associated clinical features and antibiotics of choice for antimicrobial treatment.

This woman’s ulcer is severely infected evident by significant cellulitis and systemic symptoms of fever and tachycardia. She should be started on intravenous antibiotics, and ticarcillin + clavulanate is an appropriate option after debridement and swabs.

Option B: Analgesics can be given for pain control. It is not a priority before debridement, swabs and antibiotics.

Option C: In the presence of an infection/inflammation, lateral, antero-posterior, and oblique X-ray views of both feet should be performed with the X-ray request specifying the anatomical location of the ulcer and mentioning the possibility of underlying osteomyelitis, diabetic arthropathy, and gas formation. However, it can be performed after more important steps (debridement, swabs, and antibiotics) have been taken.

Option D: MRI is indicated if osteomyelitis is suspected based on the following:
* Ulcers unresponsive to antibiotic therapy
* Probing the wound results in bone being touched
* Bone is visible in the base of the wound
* X-ray suggest bone involvement

In this scenario, MRI is not a priority in management.

28
Q

A 62-year-old diabetic man presents with a foot ulcer shown in the following photograph. On examination, he is not febrile. Debridement is perfomred, swabs for cultures are taken, and antibiotics started. Which one of the following is the most appropriate next step in management?

A. X-ray of the foot.
B. Blood culture.
C. MRI.
D. Daily wet dressing of the wound.
E. Ultrasonography.

A

A. X-ray of the foot.

A step-wise approach to management of diabetic foot ulcers includes:

1-Debridement
Management of diabetic foot ulcers starts with debridement as a very important initial step in management. It allows comprehensive evaluation of the wound, wound bed, and actual wound size. Debridement may also convert a chronic wound to an acute one with accelerated healing. Reducing the local pressure on the wound is another benefit. Debridement can be surgical or non-surgical. It must be noted, however, that in the presence of peripheral vascular disease wound debridement should be performed after comprehensive assessment of the vasculature status. Debridement in an ischemic limb is associated with complications.

2-Wound swabs
If infection is evident or suspected, deep wound swabs or needle aspiration of the exudate should be taken for bacteriological analysis before starting antibiotic treatment. If an ulcer can be probed down to bone, osteomyelitis is likely to be present (MRI is indicated).

It is usual to take a wound swab even in the absence of the clinical signs of infection or in an ulcer which is clean, does not probe to bone, is not producing large amounts of exudate, and has granulation tissue. The swab should be taken as localized and as deep as possible. If a pathogen is found without any clinical signs of infection, a topical bactericidal dressing, such as one containing nanocrystalline silver, may help clear the wound of bacteria.

3- Antibiotics
Antibiotics are indicated if there is minimal inflammation but odorous wound secretions, or in the presence of moderate to severe infection. Choices of antibiotics depend on the wound characteristic and associated symptoms. The following table summarizes antibiotic therapy for different diabetic ulcers.

4- Imaging
When clinical signs of inflammation are evident, lateral, antero-posterior, and oblique X-ray views of both feet should be obtained, with the X-ray request specifying the anatomical location of the ulcer and mentioning the possibility of underlying osteomyelitis, diabetic arthropathy and gas formation. Bone infection usually must be present for several weeks before it is detectable on plain X-ray films, so serial X-rays at one to four weeks intervals may be necessary if clinical infection fails to resolve and the initial X-ray is clear.

MRI of the affected area may be useful for differentiating infectious from non-infectious inflammatory conditions. However, if MRI facilities are not available, a surgeon should be able to decide regarding exploration and debridement of an ulcer based on clinical examination.

5- Other tests
Surgical biopsy of the affected area may be useful for collection of subsequent specimens for microbiological examination if deterioration occurs after empirical antibiotic therapy is started.
Full blood count, ESR, electrolytes, blood glucose and HbA1c, renal and liver function tests are indicated. Weekly measurement of C-reactive protein titer during treatment of a foot infection may help monitor the response to treatment.

In this patient, swabs have been taken and antibiotics started, and the most appropriate next step will be antero-posterior, lateral and oblique X-rays of both feet as mentioned earlier.

Option B: Blood cultures may be indicated if the patient had systemic manifestations such as fever and chills (absent in this patient).

Option C: MRI is indicated if osteomyelitis is suspected. A probe reaching the bone, bone being visible through the ulcer, or inadequate response to antibiotics makes osteomyelitis likely.

Option D: Daily wet dressing of the wound is the treatment of choice for non-infected ulcers, and adjuvant treatment to antibiotics for infectious wounds.

Option E: Ultrasound is not recommended for initial assessment of diabetic foot ulcers.

29
Q

A 65-year-old woman with long-standing history of type II diabetes mellitus (DM) presents with a right foot ulcer which she noticed 7 days ago. You admit her to the hospital for treatment based on the diagnosis of a diabetes-related foot ulcer (DRFU). Wound debridment is carried out. The following day, examination reveals that the ulcer is 1 cm in diameter and oozing an odorless clear fluid. Which one of the following is the most appropriate next step in management?

A. Oral amoxicillin-clavulanate + metronidazole.
B. Intravenous ticarcillin + metronidazole.
C. MRI of the foot.
D. Daily wet dressing of the wound and follow-up.
E. Wound toilet with povidone iodine.

A

D. Daily wet dressing of the wound and follow-up.

Foot ulceration is a major cause of mortality and morbidity in diabetic patients. Management of every diabetic-related foot ulcer (DRFU) starts with debridement as an essential component of wound bed preparation by removing non-viable tissue. This allows for comprehensive examination of the wound bed and assessment of actual wound size. Additionally, debridement may convert a chronic wound to an acute one with accelerated healing. Reducing the local pressure on the wound is another benefit.

Debridement can be surgical or non-surgical. Surgical sharp debridement is indicated when there is deep necrotic tissue, gangrene, drainage of a collection, or deep infections. Non-surgical debridement is generally required every 1-2 weeks to remove non-viable tissue, including the surrounding callous. Non-sharp debridement with enzymatic, autolytic, or biological preparations can be used as an adjunct to sharp debridement and can be helpful for adherent slough, or when pain limits surgical sharp debridement.

NOTE - In the presence of peripheral vascular disease, debridement should be avoided and the patient referred for specialist vasculature assessment. The only exception to this rule is the presence of an abscess, in which case debridement and drainage should be performed regardless of vascular status.

The rest of management relies upon whether the wound is infected or inflamed, and if so, the wound characteristics. The following table outlines further steps for wound management based on the presence of infection/inflammation and its severity.

This patient has a 1cm ulcer. There is no comment regarding surrounding inflammation or cellulitis to suggest infection. Moreover, the wound secretions are clear and non- infective. Management of such wounds, after initial debridement, includes deep wound swabs (if not already taken) and wound dressing and daily check to assess the healing process.

Although numerous topical medications and gels are promoted for ulcer care, relatively few have proved to be more efficacious than warm wet dressing.

NOTE - If the presence of peripheral vascular disease, dry wound dressing should be applied and the patient referred for specialist vascular assessment and care.

Option A and B: Antibiotics are used when the wound is associated with infection/inflammation Oral amoxicillin-clavulanate is indicated for ulcers with minimal infection or those without signs of infection but with odorous secretions. Parenteral antibiotics are indicated when there is marked infection. For deep or widespread infections or those associated with osteomyelitis, antibiotics such as piperacillin- tazobactam, ticarcillin-clavulanate, meropenem, ertapenem, carbapenem, moxifloxacin, or ciprofloxacin PLUS metronidazole, or a third-generation cephalosporin PLUS metronidazole are used.

Option C: MRI is indicated where osteomyelitis is suspected based on clinical or laboratory studies.

Option E: Topical antiseptics, such as povidone iodine, are toxic to the healing wound.

30
Q

Which one of the following tests is most likely to differentiate hypercalcemia caused by hyperparathyroidism and cancer?

A. PTH.
B. Calcium level.
C. Phosphate.
D. Alkaline phosphatase.
E. Vitamin D level.

A

A. PTH.

Primary hyperparathyroidism and malignancy are the most common causes of hypercalcemia, accounting for more than 90% of cases. Primary hyperthyroidism is the more common of these two.

It is usually not difficult to differentiate between them. Malignancy is often evident clinically by the time it causes hypercalcemia, and patients with hypercalcemia of malignancy have higher calcium concentrations and are more symptomatic from hypercalcemia compared to individuals with primary hyperparathyroidism.

However, it may be difficult to differentiate the two problems clinically when the presentation is less typical. As an example, some patients with occult malignancy may present with mild hypercalcemia. Alternatively, patients with hyperparathyroidism can occasionally have acute onset of severe, symptomatic hypercalcemia (parathyroid crisis). In such cases, measurement of intact PTH usually distinguishes these two conditions.

Intact PTH concentrations are generally undetectable or very low in hypercalcemia of malignancy and are elevated or high-normal in primary hyperparathyroidism.

It is uncommon for patients with hypercalcemia of malignancy to have elevated PTH levels. A suppressed PTH level is often indicative of malignancy. However, in rare cases there might be concomitant primary hyperparathyroidism or a PTH secreting tumor.

31
Q

A 30-year-old man presents to your practice with complaints of anxiety and palpitation. He mentions that he is always afraid that something bad is going to happen soon. On examination, he has a blood pressure of 160/80mmHg and heart rate of 110bpm. His palms are wet. Which one of the following is the most likely diagnosis?

A. Hyperthyroidism.
B. Panic disorder.
C. Pheochromocytoma.
D. Generalized anxiety disorder.
E. Hypothyroidism.

A

A. Hyperthyroidism.

There are some points to take into consideration in this scenario. First is the feeling of ‘always afraid that something bad is going to happen soon’. This subjective feeling also described as ‘impending doom’ often indicates the presence of anxiety and can be seen during a panic attack, in patients with pheochromocytoma, and in those with hyperthyroidism and thyrotoxicosis. Impending doom is described by patients as if something very bad is just about to happen but they do not know what it is, or as if the world is going to end, or like they are going to die of a heart attack.

Some other conditions that can cause such sensation include:
* Generalized anxiety disorder
* Temporal lobe epilepsy
* Excessive caffeine use
* Sleep deprivation
* Depression in adults
* Agoraphobia
* Thyroid storm
* Hyperthyroidism
* Hyperventilation
* Hypopituitarism
* Anaphylaxis

Increased perspiration and palpitation are features seen almost in every hypermetabolic and hypersympathetic state. Sweaty hands can be seen in episodic attacks of pheochromocytoma, hyperthyroid states, anxiety, and during a panic attack.

Pheochromocytoma, hyperthyroidism and acute panic attack can present with tachycardia and hypertension. Unless the patient is experiencing an acute panic attack right now, it does not seem to be the case. Elevated blood pressure on examination goes against generalized anxiety disorder (option D) and panic disorder (option B), making these two less likely, but still possible. Sustained elevation of blood pressure is not a typical feature of anxiety disorders.

In fact, of the options, pheochromocytoma and hyperthyroidism top the list of differentials. The most common presentation of pheochromocytoma is with episodic headache, tachycardia, hypertension, anxiety and sweating. Although some patients with pheochromocytoma appears to have sustained increased blood pressure, the absence of headache, which is one of the components of the classic triad of the disease (headache, palpitation, and diaphoresis), makes pheochromocytoma less likely. Moreover, there is no comment regarding episodic nature of the symptoms.
Having said these, hyperthyroidism would be the most likely diagnosis. Anxiety, the senses of something bad is going to happen, palpitations, sweatiness and elevated systolic blood pressure are reasonably justified by such a diagnosis.

The most frequent symptoms of hyperthyroidism are nervousness (anxiety), heat intolerance, palpitations, and fatigue and weight loss. Common signs on examination include agitation, sinus tachycardia, elevated systolic blood pressure, fine tremors and hyper-reflexia.

Hypothyroidism (option E) causes bradycardia and dry skin, and is not associated with anxiety and the sense of impending doom

32
Q

A 64-year-old woman presents to your practice with complaints of palpitation and anxiousness. She has past medical history of hypertension and ischemic heart disease. He also has hypothyroidism, for which she is currently on levothyroxine 150 mcg, daily. Based on the history and clinical findings, you suspect hyperthyroidism caused by overtreatment of her hypothyroidism. A serum thyroid stimulation hormone (TSH) confirms the diagnosis. You decrease the dose of levothyroxine to 100 mcg, daily. When you should perform a follow-up TSH level again?

A. After one day.
B. After one week.
C. After two weeks.
D. After four weeks.
E. After six months.

A

D. After four weeks.

Thyrotoxicosis is common in the Australian community and is frequently encountered in general practice.

The most frequent symptoms of thyrotoxicosis are nervousness, heat intolerance, palpitations, fatigue and weight loss despite increased appetite (weight gain occurs in 10% of patients). Common signs on examination include agitation, sinus tachycardia, fine tremor and hyper- reflexia. Elderly patients often present with nonspecific symptoms. Up to 20% of the elderly patients have atrial fibrillation (AF).

Most presentations of thyrotoxicosis are due to Graves disease, toxic multinodular goiter, toxic adenoma and thyroiditis. Graves disease is by far the most common cause of thyrotoxicosis.

Exogenous thyroid hormone is another cause of thyrotoxicosis. Exposure to excess thyroid hormone usually occurs with overtreatment of hypothyroidism and intentional ingestion in factitious disorder.

Overtreatment with thyroid hormone should always be suspected when a patient, who is on with thyroxin replacement therapy for hypothyroidism, presents with signs and symptoms of hyperthyroidism.

This patient has presented with the history and clinical findings consistent with hyperthyroidism and thyrotoxicosis. The diagnosis is established with presence of depressed TSH levels. As the most appropriate option, thyroxin should be discontinued or the dose reduced. When therapy is stopped, serum T4 concentration falls about 50% in 7 days.

Response to treatment should be monitored at 4-to 6-week intervals, and dose adjustment performed until the patient is euthyroid again.

NOTE - After starting the treatment of hypothyroidism, the first follow-up with TSH should be performed in 6-8 weeks. This is different from the above where followup after dose reduction is required.

33
Q

A 29-year-old woman presents to your GP clinic with complaints of mood swings, palpitation, and fine tremors for the past few weeks. Laboratory tests are ordered and the results are remarkable for a decreased TSH level and normal T3 and T4. Which one of the following is most likely to be the cause of this presentation?

A. Grave’s disease.
B. Pregnancy.
C. A toxic thyroid nodule.
D. Hyperthyroidism.
E. Hypothyroidism.

A

A. Grave’s disease.

The constellation of symptoms is suggestive of hyperthyroidism. The laboratory findings consistent with the diagnosis are expected to be a subnormal TSH with elevated serum T4, T3, or both. This patient has a subnormal TSH level in the presence of normal T3 and T4 values – an entity termed subclinical hyperthyroidism.

It should be noted that subclinical versus overt hyperthyroidism are only biochemical definitions because hyperthyroid symptoms may be present in those with subclinical hyperthyroidism, and absent in those with overt hyperthyroidism. In other words any patient with suboptimal TSH and symptoms of thyrotoxicosis such as palpitations, sweating, increased appetite, tremors, etc. has hyperthyroidism despite of normal T4 and T3.

The cause of subclinical hyperthyroidism are the same as the causes of overt hyperthyroidism with Graves disease being the most common cause for both.

Option C: A toxic nodule is another cause but is less common than Graves disease.

Option B: During pregnancy, production of the thyroid hormones increases by approximately 50%. As a result TSH level during pregnancy is lower than the normal non-pregnant level, but still within the normal range. The increase in T4 and T3 are due to an increase in thyroid binding globulin (TBG). Free T4 and T3 remain unchanged.

Option D: Hyperthyroidism is a general term describing a set of symptoms associate with excess thyroid hormones production that can be caused by a variety of diseases, of those Grave’s disease is the most common one.

Option E: Hypothyroidism presents with elevated TSH and decreased T3 and T4. Subclinical hypothyroidism is defined as elevated TSH beyond the normal limit but normal values of thyroid hormones.

34
Q

An 83-year-old man fell off a slippery toilet bowl in the bathroom while trying to reach toilet paper, and had his left femoral neck fractured. He was treated with open reduction and internal fixation. Today, he is being discharged. Which one of the following pieces of advice is most important to give?

A. He should start alendronate.
B. He should have a bone scan.
C. He should take low molecular weight heparin (LMWH) for 6 weeks. D. He should take warfarin for 6 months.
E. He should take supplemental calcium and vitamin D.

A

A. He should start alendronate.

Femoral neck fracture following falling off a short height is in fact a fracture following a minor trauma. Minor (low) trauma fractures are always pathologic. Osteoporosis is the most common cause of pathologic fractures. Unlike in women, osteoporosis in man is often secondary and a manifestation of an underlying cause. In men with clinical manifestations of osteoporosis e.g., low trauma fracture or with low bone mass (T-score less than -2.0), an initial evaluation for secondary causes of osteoporosis should be performed. Conditions to consider are renal or liver disease, hyperparathyroidism, Cushing’s syndrome, celiac disease, and other forms of malabsorption, or hypercalciuria.

The initial evaluation should include routine biochemical tests to for renal or hepatic disease, a full blood count (FBC), serum testosterone, calcium, alkaline phosphatase, 25-hydroxyvitamin D, and 24-hour urine calcium.

In addition to assessment for the underlying cause and treating them, if possible, patients with pathological fractures due to osteoporosis, should be started on osteoporosis-specific treatment for prevention and treatment of osteoporosis.

Bisphosphonates are (e.g., alendronate, residronate, zoledronic acid) are first-line treatment options for this purpose and the best advice to give for prevention of further fractures and bone density enhancement.

Option B: Although it is recommended that bone mineral density scan be measured, treatment can be started without a bone scan.

Option C: Patients undergoing hip fracture surgery are in the highest risk category for development of venous thromboembolism (VTE), based on the procedure itself. Without thromboprophylaxis, reported rates of VTE are high following hip surgery. This necessitates thromboprophylaxis following surgery for all patients admitted to hospital with hip fracture. The recommended regimen is low molecular weight heparin (LMWH) (first-line) or fondaparinux for up to 35 days starting 12 hours after the surgery. Although, thromboprophylaxis is more vital than alendronate, it has already been started post-operatively and continued during hospital stay, which is often 10-14 days. Six weeks of anticoagulation with LMWH is beyond the maximum duration recommended and not a correct option.

Option D: Warfarin may be less effective than LMWH, and it has the drawback of the need for INR monitoring; however, patients who wish to avoid the discomfort of a twice daily injection may be more compliant with warfarin. Six months, however, is way beyond the recommended duration of prophylaxis.

Option E: Calcium and vitamin D supplementation should be advised if dietary calcium and vitamin D are inadequate. Supplementation with calcium and vitamin D has not shown to reduce the risk of further fractures in the future.

35
Q

Daniel, a 78-year-old patient of yours, presents for consultation. He has a fall last month and had his femoral neck fractured. He underwent surgery and intramedullary nail placement. The fracture occurred when the chair leg broke and he fell off onto the ground from a height of approximately 70cm. In laboratory assessment, significant findings are a testosterone level of 8ng/dL (normal:8-50ng/dL) and vitamin D of 12mmol/L (≥75mmol/L). Serum calcium is normal, as are other laboratory parameters including liver and renal function tests. Which one of the following is the most important management to consider as the next step?

A. Alendronate 70mg, once weekly.
B. Calcium 1000mg, daily plus vitamin D 600IU, daily.
C. Calcitriol 0.25mg, daily.
D. Testosterone replacement therapy.
E. Adequate sun exposure.

A

C. Calcitriol 0.25mg, daily.

The femoral neck fracture caused by falling off a short chair, as described, is in fact a fracture following a minor (low) trauma. This is always pathologic. Osteoporosis is the most common cause of pathologic fractures. Unlike in women, osteoporosis in men is almost always secondary and a manifestation of an underlying disease. Therefore, in men with clinical manifestations of osteoporosis e.g. low trauma fracture or with low bone mass by DEXA (T-score less than -2.0), an initial evaluation for secondary causes of osteoporosis should be performed.

Conditions to consider are renal or liver disease, hyperparathyroidism, Cushing’s syndrome, celiac disease and other forms of malabsorption, or hypercalciuria.

The initial evaluation should include routine biochemical tests for renal or hepatic disease, a full blood count (FBC), serum testosterone, calcium, alkaline phosphatase, 25-hydroxyvitamin D , and 24-hour urine cortisol.

Based on the test results, Daniel is suffering from severe vitamin D deficiency and vitamin D should be replenished as the most appropriate next step in management. The treatment option of choice is calcitriol 0.25mg, daily.

Option A: Bisphosphonates are medications of choice for treatment of osteoporosis and prevention of further fractures in patients with osteoporosis, whose osteoporosis is not due to hypogonadism (low testosterone levels), or those with hypogonadism in whom testosterone replacement therapy is contraindicated. Indications are a T-score or less than -2.5 or history of pathologic fracture regardless of the T- score. The two most widely used drugs are alendronate and residronate. Alendronate should be given 70 mg, weekly (or 10mg, daily). It is, however, necessary that vitamin D be corrected to a level of above 50nmol/L before therapy with bisphosphonates is started.

Option B: This patient has normal serum calcium levels and does not need calcium supplementation now. Moreover, calcium should not be added to calcitriol therapy due to the risk of hypercalcemia. Dietary calcium intake should be maintained at 800 to 1000 mg, daily.

Option D: Testosterone replacement therapy is the treatment of choice in osteoporotic men with hypogonadism. Daniel’s testosterone level is in the lower limit normal range. Judged by his testosterone level, he is unlikely to have hypogonadism.

Option E: Adequate sun exposure should be recommended to all patients as a means of preventing vitamin D deficiency. Daniel has severe vitamin D deficiency and needs to be treated more aggressively rather than just with sun exposure.

Calcitriol = active form of Vit D.

36
Q

A previously healthy and fit 35-year-old man presents with complaints of recent-onset palpitations and feeling edgy. On examination, he looks anxious. His blood pressure is 160/80mmHg and heart rate 102 bpm. The rest of the exam is inconclusive. Which one of the following could be the most likely diagnosis?

A. Pheochromocytoma.
B. Panic attack.
C. Anxiety.
D. Hyperthyroidism.
E. Carcinoid syndrome.

A

D. Hyperthyroidism.

Pheochromocytoma, panic attack, anxiety and hyperthyroidism can all present with palpitation, anxiousness and nervousness, and palpitation.

The classic history of a patient with a pheochromocytoma (option A) includes spells of headache, palpitation, and diaphoresis,
and hypertension
. These four characteristics together are strongly suggestive of a pheochromocytoma. The spells of pheochromocytoma may vary in occurrence from monthly to several times per day, and the duration may vary from seconds to hours. Typically, they worsen with time, occurring more frequently and becoming more severe as the tumor grows. There is no comment regarding episodic nature of the symptoms in the history. Furthermore, the patient mentions no headache or excessive sweating. Considering these facts, pheochromocytoma is less likely to be the diagnosis.

Panic attack (option B) can also present with palpitations and pounding heart, anxiousness, sweating, trembling or shaking, sensation of shortness of breath, choking or smothering, chest pain or discomfort, nausea and abdominal discomfort, and dizziness. Patients with panic disorder have recurrent episodes of panic, with the fear of recurrences resulting in significant behavioral changes and worry about the consequences of the attack. They may feel they are going crazy, losing control, or dying. Feeling of impending doom (somatic concerns of death from cardiac or respiratory problems) may be a major concern of patients during an attack. Patients may seek emergency care in the emergency department. There are no signs specific for panic disorder. If the patient presents in an acute state of panic, hypertension, tachycardia, mild tachypnea, mild tremors, and cool, clammy skin might be noted. Panic attack normally lasts 20 to 30 minutes but rarely in can continue for 1 hour.

Again, there is no comment regarding episodic nature of symptoms as expected in panic disorder. Also, the patient does not appear to have been experiencing an acute attack while being examined to justify the elevated blood pressure.

Patients with anxiety disorders (option C) often experience anxiousness and excess worry. They also experience bouts of palpitation and heart pounding, but hypertension is not often an associated feature.

Carcinoid syndrome (neuroedocrine tumour) (option E) most commonly presents with diarrhea, flushing and wheezing which are absent here. In fact carcinoid syndrome is the least likely diagnosis among others.

With the given constellation of symptoms, hyperthyroidism is the best fit for this scenario and the most likely diagnosis among others.

The most frequent symptoms of hyperthyroidism are nervousness, heat intolerance, palpitations, and fatigue and weight loss. Common signs on examination include agitation, sinus tachycardia, elevated systolic blood pressure, fine tremor and hyper-reflexia. Elderly patients often present with nonspecific symptoms. Up to 20% of the elderly patients have atrial fibrillation (AF).

37
Q

A 37-year-old woman, who underwent thyroidectomy 4 days ago, presents with complaints of tingling of her hands and feet and around her mouth. Her total calcium level was 2.02 (normal 2.1- 2.6 mmol/L) on discharge. Which one of the following is the best immediate management of this patient?

A. Intravenous calcium.
B. Calcium gluconate, orally.
C. Calcium gluconate and vitamin D, orally.
D. Diazepam.
E. Calcitriol.

A

A. Intravenous calcium.

Every patient with symptomatic hypocalcemia (paresthesia of hands, feet and perioral area are sensory equivalents of tetany) with any calcium concentration below the reference and those with asymptomatic hypocalcemia, whose total calcium levels have acutely fallen below 1.9 mmol/L are recommended to urgently have calcium replaced intravenously.

10mL - 20ml of 10% calcium gluconate in 50-100 ml of Dextrose 5% should be given intravenously over 10-20 minutes with ECG monitoring. This can be repeated until the patient is asymptomatic. It should be followed with a calcium gluconate infusion. Calcium chloride can be used as an alternative to calcium gluconate, but it is more irritating to veins and should only be given via a central line.

If the case was asymptomatic hypocalcemia (with calcium above 1.9 mmol/L) or latent tetany (represented by positive Trousseau and/or Chvostek sign) oral calcium with or without vitamin D supplementation was the choice.

38
Q

A 55-year-old woman presents to your practice after noticing a clear nipple discharge from her right breast. On physical examination, no palpable mass is found in her breasts. Both nipples appear normal, however, pressure over the right areola causes discharge of clear fluid from the nipple. Which one of the following could be the most likely diagnosis?

A. Benign ductal papilloma.
B. Paget disease of the breast.
C. Ductal ectasia.
D. Intraductal breast carcinoma.
E. Breast fibroadenoma.

A

A. Benign ductal papilloma.

Nipple discharge is always an abnormal finding except in late pregnancy or the postpartum period. Based on characteristics, there are seven types of nipple discharge:

  1. Milky: white discharge; sometimes fat globules are seen under microscopy
  2. Multicolored gummous: sticky discharge
  3. Purulent: pus with white cells seen under microscopy
  4. Watery: colorless discharge
  5. Serous: faintly yellow, thin discharge
  6. Serosanguineous: thin, clear discharge with pink tint, RBCs seen under microscopy
  7. Bloody (sanguinous): pure blood

Conditions associated with each type of discharge are as follows:

  • Milky Discharge
    Galactorrhea, or nonpuerperal lactation, usually results from multiple duct discharge from both breasts. The most common cause of nonpuerperal lactation is hyperprolactinemia associated with pituitary adenomas, medications or other causes resulting in increased production of prolactin (e.g. primary hypothyroidism). In many women, galactorrhea can be idiopathic.
  • Multicolored and Sticky Discharge
    Duct ectasia or comedomastitis is the most common cause of a multicolored, sticky discharge that is commonly bilateral and usually in perimenopausal woman. It begins as a dilation of the terminal ducts with an irritating lipid fluid collection and producing an inflammatory reaction resulting in discharge from the nipple.

Duct ectasia is most frequently associated with pain, itching, and swelling in the nipple. Palpation of the areola can often reveal a tubular mass, reflecting the dilated ducts. Often a history of nipple manipulation is elicited. If the disease progresses, a mass can develop (plasma cell mastitis) that can mimic cancer. Surgery is indicated only if a mass forms or the discharge changes to serosanguinous or bloody.

  • Purulent Discharge
    In patients with acute puerperal mastitis, chronic lactation mastitis, central breast abscesses, or plasma cell mastitis, nipple discharge is purulent and usually unilateral. Breast cultures and smears may reveal a causative organism.

Abscess formation usually requires incision and drainage if appropriate antibiotics and warm compreses are not effective. It is important to remove a portion of an abscess wall for histologic study to exclude an underlying cancer associated with secondary necrosis and infection.

  • Watery, serous, serosanguinous, and bloody Discharges

Intraductal papillomas are the most common cause of these discharges, but fibrocystic disease, advanced duct ectasia, breast cancer, and vascular engorgement in near-term pregnancy can also be the causes.

In patients over 50, malignancy becomes increasingly common, especially if the discharge is unilateral and associated with a mass. Surgical exploration is mandatory in this group of patients with this type of discharge, even if cytologic and mammographic findings are negative.

Option B: Paget disease of the breast most commonly presents with a scaly, raw, vesicular, or ulcerated lesion that begins on the nipples and then spreads to the areola. Nipple discharge, if present at all, tends to be bloody rather than clear.

Option C: Ductal ectasia presents with multicolored and sticky discharge. Toothpaste like discharge is a classic description of discharge associated with duct ectasia.

Option D: With unilateral clear discharge in this woman, the most likely cause is a benign Intraductal papilloma. Although advanced duct ectasia and intraductal carcinoma are among other etiologies, they are less likely to be the cause compared with intraductal papilloma.

Option E: Fibroadenoma does not cause nipple discharge.

39
Q

A 30-year-old Australian woman presents to your practice complaining of fatigue, constipation, and weight gain for the past few months. She has no prior history of neck surgery or radiation. On examination, she has hoarseness and her skin is dry. You order thyroid function tests (TFT) that reveals an elevated serum thyroid stimulating hormone (TSH) and decreases T4. Which one of the following is the most likely cause of this presentation?

A. Thyroid cancer.
B. Post-ablative hypothyroidism.
C. An autoimmune disease.
D. Pituitary hypofunctioning.
E. Iodine deficiency.

A

C. An autoimmune disease.

The clinical findings and the history suggest hypothyroidism as the most likely diagnosis. The diagnosis is confirmed by typical laboratory findings of an elevated serum Thyroid Stimulating Hormone (TSH) and decreased T4.

Iodine deficiency remains the most common cause of hypothyroidism worldwide. However, in Australia and other iodine-replete countries, autoimmune chronic lymphocytic thyroiditis (Hashimoto’s thyroiditis) is the most common etiology.

Primary hypothyroidism refers to hypothyroidism caused by the thyroid gland problems. Primary hypothyroidism accounts for almost 95% of cases of hypothyroidism.

40
Q

A 29-year-old woman presents to your office with concerns about osteoporosis. She is an elite athlete and has a body mass index (BMI) of 15 kg/m2. She has two daughters aged 2 and 4 years. You order a bone densitometry with DEXA scan, the result of which reveals a T-score of -3 and -1.7 for the femur and vertebrae, respectively. In addition to supplemental calcium, which one of the following should be given to her?

A. Alendronate.
B. Eestrogen replacement therapy.
C. Vitamin D.
D. Weight bearing exercise.
E. Anabolic agents.

A

C. Vitamin D.

The low BMI of this woman, as well as being a professional athlete makes anorexia nervosa (AN) a very likely diagnosis. If the diagnosis is AN, it is most likely to be of restricting type, where the patients reduces weight through restricted calorie intake or excessive exercise.

Regardless of the diagnosis, one thing is certain: this woman is at increased risk of osteopenia/osteoporosis due to low body weight and prolonged menopause and the DEXA scan proves that osteoporosis of the hip and osteopenia of the vertebrae have already occured.

Such patients should be approached similar to those who have osteopenia/osteoporosis as a complication of AN. All patients with AN should be started on calcium (1200-1600mg, daily) and vitamin D. Calcium and vitamin D, will not restore the bone density, but they prevent further bone loss. For this patient, vitamin D is the best option to give in combination with calcium.

Option A: Alendronate and other bisphosphonates are first-line treatment for postmenopausal women with established osteoporosis. In women of reproductive age, bisphosphonates should not be used routinely as they are teratogenic and may remain in the body for long periods. Adverse effects of these drugs are not adequately studied in premenopausal women.

Option B: Estrogen therapy comes second after weight gain, as the most effective treatment. This woman, however, has already two children, making estrogen deficiency a less concerning feature.

Option D: Weight-bearing exercises have been shown equivocal results in bone mass restoration to date. Not to mention that this patient is already an athlete, making this option less effective for her. Furthermore, excessive exercise can be associated with more probability of traumatic fractures in a osteopenic/osteoporotic woman.

TOPIC REVIEW / READ PAGE 178
Osteoporosis is one of the significant complications of anorexia nervosa (AN) seen in at least 30% of female patients. Osteoporosis is associated with risk of fractures.

Osteoporosis in AN is characterized by increased bone resorption without concomitant increased bone formation. Trabecular bone, found in the spine and hips, is affected more than cortical bone. The most severely affected site is the lumbar spine. Bone loss occurs at a rate of 4% to 10% per year.

Multiple factors contribute to osteoporosis in AN. Peak bone mass is generally acquired during adolescence and young adulthood, which frequently coincides with the onset of AN. The peak bone mass acquired by a patient with AN depends on the time of onset and duration of the eating disorder, the degree of nutritional depletion, and changes in body composition.

The AN-associated osteoporosis is different from that found in the typical postmenopausal state. In AN, patients have low serum levels of estrogen, androgens, IGF-1, and leptin. In addition, there are high levels of cortisol and normal serum levels of calcium, vitamin D, and parathyroid hormone.

In all patients with AN longer than 6 months, a bone mineral density (BMD) should be measured with dual-energy X-ray absorptiometry (DEXA). For patients who continue to suffer episodes of AN, a follow-up test should be done every 2 years.

To date, weight gain and resumption of menstrual function have been shown to be the most effective treatment for osteoporosis in patients with AN. There is limited evidence supporting the use of hormone therapy in the form of estrogen, recombinant human insulin-like growth factor 1 (IGF-1), or both.

A systematic review identified five types of treatment that have been investigated:
Weight gain - Restoration of weight and menses is the cornerstone of treatment for AN, and is associated with improved bone mineral density in patients with AN and osteoporosis. However, it is not clear if bone loss is fully reversible.

Hormone therapy - Supplemental estrogen in the form of oral contraceptive (estrogen plus progestin) or estrogen replacement therapy has a secondary role in minimizing and perhaps reversing osteoporosis in patients with AN. There is evidence from multiple studies to support this widespread practice. This is in contrast to the unequivocal benefits seen in postmenopausal osteoporosis.

Bisphosphonates - Bisphosphonates should not be used routinely in AN. Long-term side effects in young patients are unknown, and bisphosphonates are teratogenic and can persist in the body for many years after discontinuation of treatment. However, Bisphosphonates have been used for severe osteoporosis, especially when AN was unlikely to remit in near future. This is based on the known effectiveness of bisphosphonates in decreasing bone resorption and increasing bone mineral density in osteopenic postmenopausal women and adolescent patients with osteogenesis imperfecta.

Calcium - Calcium and vitamin D are not effective in restoring bone mineral content in patients with AN and osteoporosis. In addition, studies have found no correlation between calcium intake and bone mineral density in AN, suggesting that adequate calcium and vitamin D do not prevent osteoporosis. However, calcium is necessary for bone maintenance and restoration, and is considered the standard treatment for patients with AN. The American Academy of Pediatrics and the Society for Adolescent Medicine both recommend that all adolescents ingest 1200-1500 mg/day of calcium and 800 IU of vitamin D to achieve peak bone mass.

Exercise - Generally, studies suggest very limited, if any, relationship between physical activity in patients with AN and their bone mineral density. A few observational studies, however, have found that exercise exerts a protective effect upon bone mineral density in patients with AN, consistent with the known effect that weight bearing exercise has in the general population.

Patients with osteoporosis should refrain from excessive exercise and activities that significantly increase the likelihood of fractures.
Other treatments such as anabolic medications, calcitonin, raloxifene has not been proved effective in treatment of AN-related osteoporosis.

Limited evidence suggests that recombinant human insulin-like growth factor 1 (IGF-1) is beneficial with greatest benefits in patients who received IGF-1 plus estrogen and progestin.

41
Q

Which one of the following is a feature more likely to be seen in hyperthyroidism?

A. Course tremors.
B. Pain in shoulders.
C. Narrow pulse pressure.
D. Delayed reflexes.
E. Weight gain.

A

B. Pain in shoulders.

Muscle weakness and proximal myopathy is a feature associated with hyperthyroidism and thyrotoxicosis. Another infrequent complication seen in patients with hyperthyroidism is chronic periarthritis and calcific tendinitis. Both conditions tend to occur in the shoulder, causing limitations of range of motion. The condition may lead to frozen shoulder (adhesive capsulitis).

NOTE - Unlike in hypothyroidism, joint pain is not as common in hyperthyroidism and if occurs is more related to periartiular structures such as tendons and muscles. Joint pain* is more common in hypothyroidism* and is often due to hypometabolic state leading to join effusion and swelling.

Option A: Course tremors are not a feature of hyperthyroidism, but fine tremors are. Fine tremors are best noted when the patient stretches out the fingers, and the examiner touches the finger tips with of palm of his/her hand.

Option C: Hyperthyroidism is associated with widened pulse pressure. Narrow pulse pressure is seen in hypothyroidism.

Option D: Delayed reflexes are a feature of hypothyroidism. Hyperthyroidism has hyper-reflexia as a feature.

Option E: An elevated metabolic state may lead to weight loss, despite increased appetite. In about 10% of patients, weight gain can be seen.

42
Q

Which one of the following clinical features is seen in both hypothyroidism and hyperthyroidism?

A. Premature grey hair.
B. Decreased libido.
C. Atrial fibrillation.
D. Cardiomegaly.
E. Hyperlipidemia.

A

B. Decreased libido.

The following table outlines the clinical features associated with hypothyroidism and hyperthyroidism.

Decrease libido is a presenting feature in both hyperthyroidism and hypothyroidism. Psychosis is another common feature.

43
Q

Which one of the following is NOT correct regarding oral bisphosphonates?

A. They reduce bone loss.
B. They increase the mineral density.
C. They are first-line treatment option for prevention of osteoporosis.
D. They cannot be continued more than 2 years.
E. They are first-line treatment for osteoporosis in postmenopausal women.

A

D. They cannot be continued more than 2 years.

Oral bisphosphonates are first-line medications for both prevention (prescribed under certain circumstances) and treatment of osteoporosis in post-menopausal women. They slow bone loss, improve bone mineral density and reduce fracture rates.

Oral bisphosphonates should be continued for at least 5 to 7 years, with review of the bone mineral density every 1 to 2 years.

44
Q

A 65-year-old female presents with Colles fracture of the right wrist. Further workup shows that she is osteoporotic. She has past history of breast cancer for which she was successfully treated with mastectomy and radiotherapy. Which one of the following would be the most appropriate management of her osteoporosis after taking care of the fracture?

A. Bisphosphonates.
B. Raloxifene.
C. Tamoxifen.
D. Premarin.
E. Calcium supplements.

A

A. Bisphosphonates.

Currently, bisphosphonates are first-line treatment for osteoporosis in patients with or without history of breast cancer. Bisphosphonates do not interact with estrogen receptors.

Although estrogen (premarin) (option D) has been used for prevention of osteoporosis in women, its use has been associated with increased risk of breast cancer if used beyond recommended duration of therapy (7 years). Estrogen-containing regimens are inappropriate options for patients with personal history of breast cancer.

Raloxifene (option B) is a selective estrogen receptor modulator (SERM) used for treatment of osteoporosis if bisphosphonates are contraindicated. It has the benefits of estrogen effects on bones without increasing the risk of estrogen-dependent cancers i.e. breast and endometrial cancers. As a matter of fact, raloxifene reduces the risk of breast cancer in high-risk postmenopausal women. This drug is often used as the most appropriate option for treating younger postmenopausal women with ONLY vertebral osteoporosis because it is not effective for prevention of non-vertebral fractures.

Neither raloxifene, nor tamoxifen (option C) are used as first-line medications for treatment of osteoporosis.

45
Q

A 55-year-old woman comes to your clinic because she is concerned about developing osteoporosis after menopause. Which one of the following is correct regarding prevention of osteoporosis?

A. Bisphosphonates are recommended only for women and just for primary prevention of osteoporosis.
B. Strontium ranelate is recommended only for treatment of severe osteoporosis in postmenopausal women.
C. Bisphosphonates are recommended only for women and just for treatment of osteoporosis.
D. In women with minimal-trauma fractures, bisphosphonates are contraindicated.
E. Alendronate is the second-line therapy in the treatment of postmenopausal osteoporosis.

A

B. Strontium ranelate is recommended only for treatment of severe osteoporosis in postmenopausal women.

Bisphosphonates (e.g. alendronate, residronate, zoledronic acid) can be used in both men and women, not only for primary prevention of osteoporosis, but also for treatment of established osteoporosis and prevention of more fractures. Bisphosphonates are always first-line therapy unless contraindicated.

Raloxifene is used in post-menopausal women as the second-line therapy, where bisphosphonates are contraindicated. It can be used for both prevention and treatment of osteoporosis.

Strontium ranelate is used to treat severe osteoporosis or those with minimal trauma fratures while on adequate dose of bisphosphonates. It works by increasing bone formation and decreasing bone loss. Myocardial infarction is a worrisome adverse effect that limits the use of this drug to severe cases of osteoporosis under strict care and surveillance.

46
Q

A 62-year-old woman presents to your clinic for discussing treatment options after she sustained a wrist fracture while playing tennis a few weeks ago. On DEXA scan she is found to have a T-score of -1.7. Which one of the following would be the most appropriate management?

A. Zoledronic acid.
B. Vitamin D supplements.
C. Advise that she should have more exercise.
D. Reassure that she does not need treatment at this stage.
E. Raloxifene.

A

A. Zoledronic acid.

Different guidelines and authors suggest various indications for commencement of medical therapy for osteoporosis. This sometimes leads to confusion as to whether which patients are candidates for medical therapy. Literally, osteoporosis is defined as a T score equal to or less than -2.5 on BDM.

Established osteoporosis is the preferred term for those with osteoporosis (a T-score less than -2.5 at the hip or spine), and one or more fragility fractures. Recognized fractures for this definition include Colles’, hip, vertebral and low-trauma fractures at the ankle. (While various other terms, such as ‘severe’, ‘advanced’ or ‘frank’ osteoporosis appears in reports, they have no specific meaning and should not be used).

A first fragility fracture doubles the risk of another fracture. With two previous fractures, there is 12-fold increase in the risk of consequent fractures. Because of such risk, current guidelines recommend treatment of those who have had a fracture and a T-score is below -1.5. In other words, the diagnosis of osteoporosis is not required to treat these individuals (a T-score≤-2.5 is not needed to start treatment).

All the following patients need medical treatment:

  • Those with a minimal trauma fracture (even if the T-score is not <-2.5 ; a T-score of <-1.5 and a fracture necessitate medical treatment)
  • Those 70 years old or older
  • Those currently on prolonged (at least 3 months) high dose corticosteroid treatment (at least 7.5 mg/day prednisolone or equivalent) and with a T-score of –1.5 or lower.

For others, ensuring adequate dietary calcium and vitamin D intake (or supplementation if there is adequate dietary intake) and lifestyle modification such as exercise, reducing alcohol consumption and stopping smoking.

All other patients need to be treated medically. Unless contraindicated, the first-line option for treatment of osteoporosis is always bisphosphonates. Although alendronate was considered the drug of choice, recent recommendation favors the use of intravenous zoledronic acid due to more compliance. Calcium and vitamin D should be given to all patients undergoing medical treatment regardless of the specific agent.

If the patient is vitamin D deficient, correction of vitamin D level (25-hudroxy vitamin D>50 nmol/L) is the most important step in management prior to commencement of treatment with bisphosphonates. The patient should receive vitamin D first if:

  • There is documented vitamin D deficiency based on serum 25-hydroxy vitamin levels
  • Housebound women
  • Nursing home residents
  • Women who are shrouded due to cultural reasons
47
Q

A 34-year-old woman, who is a known case of schizophrenia currently controlled on risperidone, presents to your practice with a 4-month history of amenorrhea. Physical examination is inconclusive. Among the laboratory tests, a raised prolactin level at 7845 mU/L is significant. Which one of the following is the most likely diagnosis?

A. Hyperprolactinemia due to risperidone.
B. Prolactinoma.
C. Hypothyroidism.
D. Stress-related illness.
E. Chronic renal failure.

A

B. Prolactinoma.

The signs and symptoms of this patient and the prolactin level on the test results is suggestive of hyperprolactinemia. The causes of hyperprolactinaemia are listed in the following table.

According to this table, all of the options can be underlying causes of hyperprolactinemia, but with prolactin level as high as 7845 mU/L, the most likely cause would be a prolactin-secreting pituitary adenoma (prolactinoma). A prolactin level of above 5000 mU/L is more likely to have been caused by a prolactinoma.

Although drugs such as risperidone are a potential cause of hyperprolactinemia, the prolactin levels are expected to be below 5000 mU/L. This is true about hyperprolactinemia due to stress-related illness and hypothyroidism.